Решение уравнений примеры и решение: Ваш браузер не поддерживается

Содержание

Решение линейных уравнений с примерами

Уравнение с одним неизвестным, которое после раскрытия скобок и приведения подобных членов принимает вид  

aх + b = 0, где a и b произвольные числа, называется линейным уравнением с одним неизвестным. Cегодня разберёмся, как эти линейные уравнения решать.

Например, все уравнения:

2х + 3= 7 – 0,5х;  0,3х = 0;  x/2 + 3 = 1/2 (х – 2) — линейные.

Значение неизвестного, обращающее уравнение в верное равенство называется решением или корнем уравнения.

Например, если в уравнении 3х + 7 = 13 вместо неизвестного х подставить число 2 , то получим верное равенство 3· 2 +7 = 13. Значит, значение х = 2 есть решение или корень уравнения.

А значение х = 3 не обращает  уравнение  3х + 7 = 13 в верное равенство, так  как  3· 2 +7 ≠ 13. Значит, значение х = 3 не является решением или корнем уравнения.

Решение любых линейных уравнений сводится к решению уравнений вида

aх + b = 0.

Перенесем свободный член из левой части уравнения в правую, изменив при этом знак перед b на противоположный, получим

aх = ‒ b.

Если a ≠ 0, то х = ‒ b/a .

Пример 1. Решите уравнение 3х + 2 =11.

Перенесем 2 из левой части уравнения в правую, изменив при этом знак перед 2 на противоположный, получим 
3х = 11 – 2.

Выполним вычитание, тогда
3х = 9.

Чтобы найти х надо разделить произведение на известный множитель, то есть     
х = 9 : 3.

Значит, значение х = 3 является  решением или корнем уравнения.

Ответ: х = 3.

Если а = 0 и b = 0, то получим уравнение  0х = 0. Это уравнение имеет бесконечно много  решений, так как при умножении любого числа на 0 мы получаем 0,но b тоже равно 0. Решением этого уравнения  является любое число.

Пример 2. Решите уравнение 5(х – 3) + 2 = 3 (х – 4) + 2х ‒ 1.

Раскроем скобки:
5х – 15 + 2 = 3х – 12 + 2х ‒ 1.

Сгруппируем  в левой части члены, содержащие неизвестные, а в правой ‒ свободные члены:
5х – 3х ‒ 2х =  – 12  ‒ 1 + 15 ‒ 2.

Приведем подобные члены:
0х = 0.

Ответ: х —  любое число.

Если а = 0 и b ≠ 0, то получим уравнение  0х = — b. Это уравнение решений не имеет, так как при умножении любого числа на 0 мы получаем 0, но  b ≠ 0 .

Пример 3. Решите уравнение х + 8 = х + 5.

Сгруппируем  в левой части члены, содержащие неизвестные, а в правой ‒ свободные члены:
х – х = 5 ‒ 8.

Приведем подобные члены: 
0х = ‒ 3.

Ответ: нет решений.

На рисунке 1 изображена схема решения линейного уравнения

Составим общую схему решения уравнений с одной переменной. Рассмотрим решение примера 4.

Пример 4. Пусть надо решить уравнение 

1) Умножим все члены уравнения на наименьшее общее кратное знаменателей, равное 12.

2) После сокращения получим
4 (х – 4) + 3·2 (х + 1) ‒ 12 = 6·5 (х – 3) + 24х – 2 (11х + 43)

3) Чтобы отделить члены, содержащие неизвестные и свободные члены, раскроем скобки:
4х – 16 + 6х + 6 – 12 = 30х – 90 + 24х – 22х – 86 .

4) Сгруппируем в одной части члены, содержащие неизвестные, а в другой – свободные члены:
4х + 6х – 30х – 24х + 22х = ‒ 90 – 86 + 16 – 6 + 12.

5) Приведем подобные члены:
‒ 22х = ‒ 154.

6) Разделим на  – 22 , Получим
х = 7.

Как видим, корень уравнения равен семи.

Вообще такие уравнения можно решать по следующей схеме:

а) привести уравнение к целому виду;

б) раскрыть скобки;

в) сгруппировать члены, содержащие неизвестное, в одной части уравнения, а свободные члены ‒ в другой;

г) привести подобные члены;

д) решить уравнение вида aх = b,которое получили после приведения подобных членов.

Однако эта схема не обязательна для всякого уравнения. При решении многих более простых уравнений приходится начинать не с первого, а со второго (Пример. 2),  третьего (Пример. 1, 3) и даже с пятого этапа, как в примере 5.

Пример 5. Решите уравнение 2х = 1/4.

Находим неизвестное  х = 1/4 : 2,
х = 1/8
.

Рассмотрим решение некоторых линейных уравнений, встречающихся на основном государственном экзамене.

Пример 6. Решите уравнение 2 (х + 3) = 5 – 6х.

Решение

2х + 6 = 5 – 6х

2х + 6х = 5 – 6

8х = ‒1

х = ‒1 : 8

х = ‒ 0, 125

Ответ: ‒ 0, 125

Пример 7. Решите уравнение – 6 (5 – 3х) = 8х – 7.

Решение

– 30 + 18х = 8х – 7

18х  – 8х =  – 7 +30

10х = 23

х = 23 : 10

х = 2,3

Ответ: 2,3

Пример 8. Решите уравнение

 

Решение:

3(3х – 4) = 4 · 7х + 24

9х – 12 = 28х + 24

9х – 28х = 24 + 12

-19х = 36

х = 36 : (-19)

х = — 36/19

Ответ: — . 

Пример 9. Найдите f(6), если f (x + 2) = 37-х

Решение

Так как надо найти f(6), а нам известно f (x + 2),
то х + 2 = 6.

Решаем линейное уравнение х + 2 = 6,
получаем х = 6 – 2, х = 4.

Если х = 4, тогда
f(6) = 37-4 = 33 = 27

Ответ: 27.

Если у Вас остались вопросы, есть желание разобраться с решением уравнений более основательно, записывайтесь на мои уроки в РАСПИСАНИИ. Буду рада Вам помочь!

Также TutorOnline советует посмотреть новый видеоурок от нашего репетитора Ольги Александровны, который поможет разобраться как с линейными уравнениями, так и с другими.

© blog.tutoronline.ru, при полном или частичном копировании материала ссылка на первоисточник обязательна.

Урок 49. уравнения. методы решения уравнений — Алгебра и начала математического анализа — 11 класс

Алгебра и начала математического анализа, 11 класс

Урок №49. Уравнения. Методы решения уравнений.

Перечень вопросов, рассматриваемых в теме:

  • Методы решения уравнений.
  • Применение методов решения к уравнениям различного вида.
  • Примеры решения задач государственной итоговой аттестации

Глоссарий по теме

Уравнение. Пусть заданы функции f(x) и g(x). Если относительно равенства поставлена задача отыскания всех значений переменной, при которых получается верное числовое равенство, то говорят, что задано уравнение с одной переменной.

Основная литература

Колягин Ю.М., Ткачёва М.В., Фёдорова Н.Е., Шабунин М.И. Под ред. А.Б. Жижченко. Алгебра и начала математического анализа. 11 класс: учеб. для общеобразоват. учреждений: базовый и профил. Уровни. – 2-е изд. – М.: Просвещение, 2010.

Открытые электронные ресурсы:

Решу ЕГЭ образовательный портал для подготовки к экзаменам https://ege.sdamgia.ru/.

Открытый банк заданий ЕГЭ ФИПИ, Элементы комбинаторики, статистики и теории вероятностей, базовый уровень. Элементы комбинаторики, статистики и теории вероятностей. Базовый уровень. http://ege.fipi.ru/.

Теоретический материал для самостоятельного изучения

Основные методы решения уравнений

Метод разложения на множители

Рассмотрим пример.

Решить уравнение:

ООУ:

Преобразуем обе части уравнения

Это уравнение равносильно совокупности двух уравнений

или

Первое уравнение

имеет множество корней

Второе уравнение

равносильно и его корни

Ответ:

Метод замены переменной

Рассмотрим пример.

ООУ:

Так как в уравнении присутствует повторяющееся выражение, введем новую переменную

и получи уравнение

, корни которого

Возвращаемся к первоначальной переменной

или

Ответ:

Метод решения однородных уравнений.

Рассмотрим пример

Решить уравнение:

ООУ: x – любое действительное число

Все слагаемые в правой части уравнения имеют равные степени, поэтому разделим обе части уравнения на и получим

.

Решаем полученное уравнение методом замены переменной

или

Ответ: 1; 2

Итак, можно сделать следующие выводы. Наличие в уравнении повторяющихся элементов позволяет сделать предположение, что в его решении можно применить метод замены переменной. Наличие общих множителей выводит на применение метода разложение на множители. Если же в одной из частей уравнения стоит однородный многочлен, то применяем метод решения однородных уравнений.

Примеры и разбор решения заданий тренировочного модуля

Пример 1.

Решите уравнение

Выберите ответ из предложенных.

Варианты ответов:

  1. 10
  2. -10
  3. 100
  4. -100
  5. 1000
  6. -1000

Решение

ООУ:

Преобразуем левую часть уравнения

Введем новую переменную

Получим уравнение

Возвращаемся к первоначальной переменной

Ответ: — 1000

Пример 2.

Решите уравнение

Выберите корень из списка:

Решение:

ООУ:

Возведем обе части уравнения в квадрат

Повторно возведем в квадрат при условии

Корни этого уравнения

Учитывая все ограничения, получаем ответ .

Решение линейных уравнений с примерами. Уравнения онлайн Примеры уравнений 5

Уравнение с одним неизвестным, которое после раскрытия скобок и приведения подобных членов принимает вид

aх + b = 0 , где a и b произвольные числа, называется линейным уравнением с одним неизвестным. Cегодня разберёмся, как эти линейные уравнения решать.

Например, все уравнения:

2х + 3= 7 – 0,5х; 0,3х = 0; x/2 + 3 = 1/2 (х – 2) — линейные.

Значение неизвестного, обращающее уравнение в верное равенство называется решением или корнем уравнения .

Например, если в уравнении 3х + 7 = 13 вместо неизвестного х подставить число 2 , то получим верное равенство 3· 2 +7 = 13. Значит, значение х = 2 есть решение или корень уравнения.

А значение х = 3 не обращает уравнение 3х + 7 = 13 в верное равенство, так как 3· 2 +7 ≠ 13. Значит, значение х = 3 не является решением или корнем уравнения.

Решение любых линейных уравнений сводится к решению уравнений вида

aх + b = 0.

Перенесем свободный член из левой части уравнения в правую, изменив при этом знак перед b на противоположный, получим

Если a ≠ 0, то х = ‒ b/a .

Пример 1. Решите уравнение 3х + 2 =11.

Перенесем 2 из левой части уравнения в правую, изменив при этом знак перед 2 на противоположный, получим
3х = 11 – 2.

Выполним вычитание, тогда
3х = 9.

Чтобы найти х надо разделить произведение на известный множитель, то есть
х = 9: 3.

Значит, значение х = 3 является решением или корнем уравнения.

Ответ: х = 3 .

Если а = 0 и b = 0 , то получим уравнение 0х = 0. Это уравнение имеет бесконечно много решений, так как при умножении любого числа на 0 мы получаем 0,но b тоже равно 0. Решением этого уравнения является любое число.

Пример 2. Решите уравнение 5(х – 3) + 2 = 3 (х – 4) + 2х ‒ 1.

Раскроем скобки:
5х – 15 + 2 = 3х – 12 + 2х ‒ 1.


5х – 3х ‒ 2х = – 12 ‒ 1 + 15 ‒ 2.

Приведем подобные члены:
0х = 0.

Ответ: х — любое число .

Если а = 0 и b ≠ 0 , то получим уравнение 0х = — b. Это уравнение решений не имеет, так как при умножении любого числа на 0 мы получаем 0, но b ≠ 0 .

Пример 3. Решите уравнение х + 8 = х + 5.

Сгруппируем в левой части члены, содержащие неизвестные, а в правой ‒ свободные члены:
х – х = 5 ‒ 8.

Приведем подобные члены:
0х = ‒ 3.

Ответ: нет решений.

На рисунке 1 изображена схема решения линейного уравнения

Составим общую схему решения уравнений с одной переменной. Рассмотрим решение примера 4.

Пример 4. Пусть надо решить уравнение

1) Умножим все члены уравнения на наименьшее общее кратное знаменателей, равное 12.

2) После сокращения получим
4 (х – 4) + 3·2 (х + 1) ‒ 12 = 6·5 (х – 3) + 24х – 2 (11х + 43)

3) Чтобы отделить члены, содержащие неизвестные и свободные члены, раскроем скобки:
4х – 16 + 6х + 6 – 12 = 30х – 90 + 24х – 22х – 86 .

4) Сгруппируем в одной части члены, содержащие неизвестные, а в другой – свободные члены:
4х + 6х – 30х – 24х + 22х = ‒ 90 – 86 + 16 – 6 + 12.

5) Приведем подобные члены:
‒ 22х = ‒ 154.

6) Разделим на – 22 , Получим
х = 7.

Как видим, корень уравнения равен семи.

Вообще такие уравнения можно решать по следующей схеме :

а) привести уравнение к целому виду;

б) раскрыть скобки;

в) сгруппировать члены, содержащие неизвестное, в одной части уравнения, а свободные члены ‒ в другой;

г) привести подобные члены;

д) решить уравнение вида aх = b,которое получили после приведения подобных членов.

Однако эта схема не обязательна для всякого уравнения. При решении многих более простых уравнений приходится начинать не с первого, а со второго (Пример. 2 ), третьего (Пример. 1, 3 ) и даже с пятого этапа, как в примере 5.

Пример 5. Решите уравнение 2х = 1/4.

Находим неизвестное х = 1/4: 2,
х = 1/8
.

Рассмотрим решение некоторых линейных уравнений, встречающихся на основном государственном экзамене.

Пример 6. Решите уравнение 2 (х + 3) = 5 – 6х.

2х + 6 = 5 – 6х

2х + 6х = 5 – 6

Ответ: ‒ 0, 125

Пример 7. Решите уравнение – 6 (5 – 3х) = 8х – 7.

– 30 + 18х = 8х – 7

18х – 8х = – 7 +30

Ответ: 2,3

Пример 8. Решите уравнение

3(3х – 4) = 4 · 7х + 24

9х – 12 = 28х + 24

9х – 28х = 24 + 12

Пример 9. Найдите f(6), если f (x + 2) = 3 7-х

Решение

Так как надо найти f(6), а нам известно f (x + 2),
то х + 2 = 6.

Решаем линейное уравнение х + 2 = 6,
получаем х = 6 – 2, х = 4.

Если х = 4, тогда
f(6) = 3 7-4 = 3 3 = 27

Ответ: 27.

Если у Вас остались вопросы, есть желание разобраться с решением уравнений более основательно, записывайтесь на мои уроки в РАСПИСАНИИ . Буду рада Вам помочь!

Также TutorOnline советует посмотреть новый видеоурок от нашего репетитора Ольги Александровны, который поможет разобраться как с линейными уравнениями, так и с другими.

сайт, при полном или частичном копировании материала ссылка на первоисточник обязательна.

Решаем дробно-рациональное уравнение 5/х = 100. Данное уравнение можно решить двумя способами. Давайте рассмотрим каждый из них.

План решения уравнения 5/x = 100

  • найдем область допустимых значений для заданного уравнения;
  • первый способ решения уравнения рассмотрев его как на пропорцию;
  • второй способ решения уравнения, находя неизвестный делитель.

Находим неизвестный член пропорции

Сначала найдем ОДЗ уравнения. В левой части уравнения присутствует знак дроби и он равносилен знаку деления. Известно, что на ноль делить нельзя. Значит из ОДЗ мы должны исключить значения обращающие знаменатель в ноль.

ОДЗ: x принадлежит R \ {0}.

Теперь посмотрим на наше уравнение как на пропорцию.

Основное свойство пропорции.

Произведение крайних членов пропорции равно произведению ее средних членов.

Для пропорции a: b = c: d или a/b = c/d основное свойство записывается так: a · d = b · c.

Применим его и получим линейное уравнение:

100 * x = 5 * 1;

Разделим на 100 обе части уравнения, тем самым избавимся от коэффициента перед переменной х:

Находим неизвестный делитель

Посмотрим на уравнение как на частное. Где делимое равно 5, делитель x, а результат деления — частное равно 100.

Вспомним правило как найти неизвестный делитель — нужно делимое разделить на частное.

Найденный корень принадлежит ОДЗ уравнения.

Проверим найденное решение уравнения. Для этого подставим найденные корень в исходное уравнение и произведем вычисления:

Решение найдено верно.

Одним из самых важных навыков при поступлении в 5 класс является умение решать простейшие уравнения. Так как 5 класс ещё не так далек от начальной школы, то и видов уравнений, которые может решать ученик не так уж и много. Мы познакомим Вас со всеми основными видами уравнений, которые необходимо уметь решать, если Вы хотите поступить в физико-математическую школу .

1 тип: «луковичные»
Это уравнения, которые почти со вероятностью встретятся Вам при поступлении в любую школу или кружок 5 класса как отдельное задание. Их легко отличить от других: в них переменная присутствует только 1 раз. Например, или .
Решаются они очень просто: необходимо просто «добраться» до неизвестной, постепенно «снимая» всё лишнее, что окружает её — как будто почистить луковицу — отсюда и такое название. Для решения достаточно помнить несколько правил из второго класса. Перечислим их все:

Сложение

  1. слагаемое1 + слагаемое2 = сумма
  2. слагаемое1 = сумма — слагаемое2
  3. слагаемое2 = сумма — слагаемое1

Вычитание

  1. уменьшаемое — вычитаемое = разность
  2. уменьшаемое = вычитаемое + разность
  3. вычитаемое = уменьшаемое — разность

Умножение

  1. множитель1 * множитель2 = произведение
  2. множитель1 = произведение: множитель2
  3. множитель2 = произведение: множитель1

Деление

  1. делимое: делитель = частное
  2. делимое = делитель * частное
  3. делитель = делимое: частное

Разберём на примере, как применять данные правила.

Заметим, что мы делим на и получаем . В этой ситуации мы знаем делитель и частное. Чтобы найти делимое, нужно делитель умножить на частное:

Мы стали немного ближе к самому . Теперь мы видим, что к прибавляется и получается . Значит, чтобы найти одно из слагаемых, нужно из суммы вычесть известное слагаемое:

И ещё один «слой» снят с неизвестной! Теперь мы видим ситуацию с известным значением произведения () и одним известным множителем ().

Теперь ситуация «уменьшаемое — вычитаемое = разность»

И последний шаг — известное произведение () и один из множителей ()

2 тип: уравнения со скобками
Уравнения данного типа чаще всего встречаются в задачах — именно к ним сводится 90% всех задач для поступления в 5 класс . В отличие от «луковичных уравнений» переменная здесь может встретиться несколько раз, поэтому решить её методами из предыдущего пункта невозможно. Типичные уравнения: или
Основная трудность — это правильно раскрыть скобки. После того, как удалось это верно сделать, следует привести подобные слагаемые (числа к числам, переменные к переменным), а после этого мы получаем самое простое «луковичное уравнение» , которое умеем решать. Но обо всём по-порядку.

Раскрытие скобок . Мы приведём несколько правил, которыми следует пользоваться в данном случае. Но, как показывает практика, верно раскрывать скобки ученик начинает только после 70-80 прорешанных задач. Основное правило таково: любой множитель, стоящий за скобками необходимо умножить на каждое слагаемое внутри скобок. А минус, стоящий перед скобкой, меняет знак всех выражений, что стоят внутри. Итак, основные правила раскрытия:



Приведение подобных . Здесь всё гораздо легче: Вам необходимо путём переноса слагаемых через знак равенства добиться того, чтобы с одной стороны стояли только слагаемые с неизвестной, а с другой — только числа. Основное правило таково: каждое слагаемое, переносимое через , меняет свой знак — если оно было с ,то станет с , и наоборот. После успешного переноса необходимо сосчитать итоговое количество неизвестных, итоговое число стоящее с другой стороны равенства, нежели переменные, и решить простое «луковичное уравнение» .

Приложение

Решение любого типа уравнений онлайн на сайт для закрепления изученного материала студентами и школьниками.. Решение уравнений онлайн. Уравнения онлайн. Различают алгебраические, параметрические, трансцендентные, функциональные, дифференциальные и другие виды уравнений.. Некоторые классы уравнений имеют аналитические решения, которые удобны тем, что не только дают точное значение корня, а позволяют записать решение в виде формулы, в которую могут входить параметры. Аналитические выражения позволяют не только вычислить корни, а провести анализ их существования и их количества в зависимости от значений параметров, что часто бывает даже важнее для практического применения, чем конкретные значения корней. Решение уравнений онлайн.. Уравнения онлайн. Решение уравнения — задача по нахождению таких значений аргументов, при которых это равенство достигается. На возможные значения аргументов могут быть наложены дополнительные условия (целочисленности, вещественности и т. д.). Решение уравнений онлайн.. Уравнения онлайн. Вы сможете решить уравнение онлайн моментально и с высокой точностью результата. Аргументы заданных функций (иногда называются «переменными») в случае уравнения называются «неизвестными». Значения неизвестных, при которых это равенство достигается, называются решениями или корнями данного уравнения. Про корни говорят, что они удовлетворяют данному уравнению. Решить уравнение онлайн означает найти множество всех его решений (корней) или доказать, что корней нет. Решение уравнений онлайн.. Уравнения онлайн. Равносильными или эквивалентными называются уравнения, множества корней которых совпадают. Равносильными также считаются уравнения, которые не имеют корней. Эквивалентность уравнений имеет свойство симметричности: если одно уравнение эквивалентно другому, то второе уравнение эквивалентно первому. Эквивалентность уравнений имеет свойство транзитивности: если одно уравнение эквивалентно другому, а второе эквивалентно третьему, то первое уравнение эквивалентно третьему. Свойство эквивалентности уравнений позволяет проводить с ними преобразования, на которых основываются методы их решения. Решение уравнений онлайн.. Уравнения онлайн. Сайт позволит решить уравнение онлайн. К уравнениям, для которых известны аналитические решения, относятся алгебраические уравнения, не выше четвёртой степени: линейное уравнение, квадратное уравнение, кубическое уравнение и уравнение четвёртой степени. Алгебраические уравнения высших степеней в общем случае аналитического решения не имеют, хотя некоторые из них можно свести к уравнениям низших степеней. Уравнения, в которые входят трансцендентные функции называются трансцендентными. Среди них аналитические решения известны для некоторых тригонометрических уравнений, поскольку нули тригонометрических функций хорошо известны. В общем случае, когда аналитического решения найти не удаётся, применяют численные методы. Численные методы не дают точного решения, а только позволяют сузить интервал, в котором лежит корень, до определённого заранее заданного значения. Решение уравнений онлайн.. Уравнения онлайн.. Вместо уравнения онлайн мы представим, как то же самое выражение образует линейную зависимость и не только по прямой касательной, но и в самой точке перегиба графика. Этот метод незаменим во все времена изучения предмета. Часто бывает, что решение уравнений приближается к итоговому значению посредством бесконечных чисел и записи векторов. Проверить начальные данные необходимо и в этом суть задания. Иначе локальное условие преобразуется в формулу. Инверсия по прямой от заданной функции, которую вычислит калькулятор уравнений без особой задержки в исполнении, взаимозачету послужит привилегия пространства. Речь пойдет о студентах успеваемости в научной среде. Впрочем, как и все вышесказанное, нам поможет в процессе нахождения и когда вы решите уравнение полностью, то полученный ответ сохраните на концах отрезка прямой. Линии в пространстве пересекаются в точке и эта точка называется пересекаемой линиями. Обозначен интервал на прямой как задано ранее. Высший пост на изучение математики будет опубликован. Назначить значению аргумента от параметрически заданной поверхности и решить уравнение онлайн сможет обозначить принципы продуктивного обращения к функции. Лента Мебиуса, или как её называет бесконечностью, выглядит в форме восьмерки. Это односторонняя поверхность, а не двухсторонняя. По принципу общеизвестному всем мы объективно примем линейные уравнения за базовое обозначение как есть и в области исследования. Лишь два значения последовательно заданных аргументов способны выявить направление вектора. Предположить, что иное решение уравнений онлайн гораздо более, чем просто его решение, обозначает получение на выходе полноценного варианта инварианта. Без комплексного подхода студентам сложно обучиться данному материалу. По-прежнему для каждого особого случая наш удобный и умный калькулятор уравнений онлайн поможет всем в непростую минуту, ведь достаточно лишь указать вводные параметры и система сама рассчитает ответ. Перед тем, как начать вводить данные, нам понадобится инструмент ввода, что можно сделать без особых затруднений. Номер каждой ответной оценки будет квадратное уравнение приводить к нашим выводам, но этого сделать не так просто, потому что легко доказать обратное. Теория, в силу своих особенностей, не подкреплена практическими знаниями. Увидеть калькулятор дробей на стадии опубликования ответа, задача в математике не из легких, поскольку альтернатива записи числа на множестве способствует увеличению роста функции. Впрочем, не сказать про обучение студентов было бы некорректным, поэтому выскажем каждый столько, сколько этого необходимо сделать. Раньше найденное кубическое уравнение по праву будет принадлежать области определения, и содержать в себе пространство числовых значений, а также символьных переменных. Выучив или зазубрив теорему, наши студенты проявят себя только с лучшей стороны, и мы за них будем рады. В отличие от множества пересечений полей, наши уравнения онлайн описываются плоскостью движения по перемножению двух и трех числовых объединенных линий. Множество в математике определяется не однозначно. Лучшее, по мнению студентов, решение — это доведенная до конца запись выражения. Как было сказано научным языком, не входит абстракция символьных выражений в положение вещей, но решение уравнений дает однозначный результат во всех известных случаях. Продолжительность занятия преподавателя складывается из потребностей в этом предложении. Анализ показал как необходимость всех вычислительных приемов во многих сферах, и абсолютно ясно, что калькулятор уравнений незаменимый инструментарий в одаренных руках студента. Лояльный подход к изучению математики обуславливает важность взглядов разных направленностей. Хотите обозначить одну из ключевых теорем и решите уравнение так, в зависимости от ответа которого будет стоять дальнейшая потребность в его применении. Аналитика в данной области набирает все мощный оборот. Начнем с начала и выведем формулу. Пробив уровень возрастания функции, линия по касательной в точке перегиба обязательно приведет к тому, что решить уравнение онлайн будет одним из главных аспектов в построении того самого графика от аргумента функции. Любительский подход имеет право быть применен, если данное условие не противоречит выводам студентов. На задний план выводится именно та подзадача, которая ставит анализ математических условий как линейные уравнения в существующей области определения объекта. Взаимозачет по направлению ортогональности взаимоуменьшает преимущество одинокого абсолютного значения. По модулю решение уравнений онлайн дает столько же решений, если раскрыть скобки сначала со знаком плюс, а затем со знаком минус. В таком случае решений найдется в два раза больше, и результат будет точнее. Стабильный и правильный калькулятор уравнений онлайн есть успех в достижении намеченной цели в поставленной преподавателем задаче. Нужный метод выбрать представляется возможным благодаря существенным отличиям взглядов великих ученых. Полученное квадратное уравнение описывает кривую линий так называемую параболу, а знак определит ее выпуклость в квадратной системе координат. Из уравнения получим и дискриминант, и сами корни по теореме Виета. Представить выражение в виде правильной или неправильной дроби и применить калькулятор дробей необходимо на первом этапе. В зависимости от этого будет складываться план дальнейших наших вычислений. Математика при теоретическом подходе пригодится на каждом этапе. Результат обязательно представим как кубическое уравнение, потому что его корни скроем именно в этом выражении, для того, чтобы упростить задачу учащемуся в ВУЗе. Любые методы хороши, если они пригодны к поверхностному анализу. Лишние арифметические действия не приведут к погрешности вычислений. С заданной точностью определит ответ. Используя решение уравнений, скажем прямо — найти независимую переменную от заданной функции не так-то просто, особенно в период изучения параллельных линий на бесконечности. В виду исключения необходимость очень очевидна. Разность полярностей однозначна. Из опыта преподавания в институтах наш преподаватель вынес главный урок, на котором были изучены уравнения онлайн в полном математическом смысле. Здесь речь шла о высших усилиях и особых навыках применения теории. В пользу наших выводов не стоит глядеть сквозь призму. До позднего времени считалось, что замкнутое множество стремительно возрастает по области как есть и решение уравнений просто необходимо исследовать. На первом этапе мы не рассмотрели все возможные варианты, но такой подход обоснован как никогда. Лишние действия со скобками оправдывают некоторые продвижения по осям ординат и абсцисс, чего нельзя не заметить невооруженным глазом. В смысле обширного пропорционального возрастания функции есть точка перегиба. В лишний раз докажем как необходимое условие будет применяться на всем промежутке убывания той или иной нисходящей позиции вектора. В условиях замкнутого пространства мы выберем переменную из начального блока нашего скрипта. За отсутствие главного момента силы отвечает система, построенная как базис по трем векторам. Однако калькулятор уравнений вывел, и помогло в нахождении всех членов построенного уравнения, как над поверхностью, так и вдоль параллельных линий. Вокруг начальной точки опишем некую окружность. Таким образом, мы начнем продвигаться вверх по линиям сечений, и касательная опишет окружность по всей ее длине, в результате получим кривую, которая называется эвольвентой. Кстати расскажем об этой кривой немного истории. Дело в том, что исторически в математике не было понятия самой математики в чистом понимании как сегодня. Раньше все ученые занимались одним общим делом, то есть наукой. Позже через несколько столетий, когда научный мир наполнился колоссальным объемом информации, человечество все-таки выделило множество дисциплин. Они до сих пор остались неизменными. И все же каждый год ученые всего мира пытаются доказать, что наука безгранична, и вы не решите уравнение, если не будете обладать знаниями в области естественных наук. Окончательно поставить точку не может быть возможным. Об этом размышлять также бессмысленно, как согревать воздух на улице. Найдем интервал, на котором аргумент при положительном своем значении определит модуль значения в резко возрастающем направлении. Реакция поможет отыскать как минимум три решения, но необходимо будет проверить их. Начнем с того, что нам понадобиться решить уравнение онлайн с помощью уникального сервиса нашего сайта. Введем обе части заданного уравнения, нажмем на кнопу «РЕШИТЬ» и получим в течение всего нескольких секунд точный ответ. В особых случаях возьмем книгу по математике и перепроверим наш ответ, а именно посмотрим только ответ и станет все ясно. Вылетит одинаковый проект по искусственному избыточному параллелепипеду. Есть параллелограмм со своими параллельными сторонами, и он объясняет множество принципов и подходов к изучению пространственного отношения восходящего процесса накопления полого пространства в формулах натурального вида. Неоднозначные линейные уравнения показывают зависимость искомой переменной с нашим общим на данный момент времени решением и надо как-то вывести и привести неправильную дробь к нетривиальному случаю. На прямой отметим десять точек и проведем через каждую точку кривую в заданном направлении, и выпуклостью вверх. Без особых трудностей наш калькулятор уравнений представит в таком виде выражение, что его проверка на валидность правил будет очевидна даже в начале записи. Система особых представлений устойчивости для математиков на первом месте, если иного не предусмотрено формулой. На это мы ответим подробным представление доклада на тему изоморфного состояния пластичной системы тел и решение уравнений онлайн опишет движение каждой материальной точки в этой системе. На уровне углубленного исследования понадобится подробно выяснить вопрос об инверсиях как минимум нижнего слоя пространства. По возрастанию на участке разрыва функции мы применим общий метод великолепного исследователя, кстати, нашего земляка, и расскажем ниже о поведении плоскости. В силу сильных характеристик аналитически заданной функции, мы используем только калькулятор уравнений онлайн по назначению в выведенных пределах полномочий. Рассуждая далее, остановим свой обзор на однородности самого уравнения, то есть правая его часть приравнена к нулю. Лишний раз удостоверимся в правильности принятого нами решения по математике. Во избежание получения тривиального решения, внесем некоторые корректировки в начальные условия по задаче на условную устойчивость системы. Составим квадратное уравнение, для которого выпишем по известной всем формуле две записи и найдем отрицательные корни. Если один корень на пять единиц превосходит второй и третий корни, то внесением правок в главный аргумент мы тем самым искажаем начальные условия подзадачи. По своей сути нечто необычное в математике можно всегда описать с точностью до сотых значений положительного числа. В несколько раз калькулятор дробей превосходит свои аналоги на подобных ресурсах в самый лучший момент нагрузки сервера. По поверхности растущего по оси ординат вектора скорости начертим семь линий, изогнутых в противоположные друг другу направления. Соизмеримость назначенного аргумента функции опережает показания счетчика восстановительного баланса. В математике этот феномен представим через кубическое уравнение с мнимыми коэффициентами, а также в биполярном прогрессе убывания линий. Критические точки перепада температуры во много своем значении и продвижении описывают процесс разложения сложной дробной функции на множители. Если вам скажут решите уравнение, не спешите это делать сию минуту, однозначно сначала оцените весь план действий, а уже потом принимайте правильный подход. Польза будет непременно. Легкость в работе очевидна, и в математике то же самое. Решить уравнение онлайн. Все уравнения онлайн представляют собой определенного вида запись из чисел или параметров и переменной, которую нужно определить. Вычислить эту самую переменную, то есть найти конкретные значения или интервалы множества значений, при которых будет выполняться тождество. Напрямую зависят условия начальные и конечные. В общее решение уравнений как правило входят некоторые переменные и константы, задавая которые, мы получим целые семейства решений для данной постановки задачи. В целом это оправдывает вкладываемые усилия по направлению возрастания функциональности пространственного куба со стороной равной 100 сантиметрам. Применить теорему или лемму можно на любом этапе построения ответа. Сайт постепенно выдает калькулятор уравнений при необходимости на любом интервале суммирования произведений показать наименьшее значение. В половине случаев такой шар как полый, не в большей степени отвечает требованиям постановки промежуточного ответа. По крайней мере на оси ординат в направлении убывания векторного представления эта пропорция несомненно будет являться оптимальнее предыдущего выражения. В час, когда по линейным функциям будет проведен полный точечный анализ, мы, по сути, соберем воедино все наши комплексные числа и биполярные пространства плоскостной. Подставив в полученное выражение переменную, вы решите уравнение поэтапно и с высокой точностью дадите максимально развернутый ответ. Лишний раз проверить свои действия в математике будет хорошим тоном со стороны учащегося студента. Пропорция в соотношении дробей зафиксировала целостность результата по всем важным направлениям деятельности нулевого вектора. Тривиальность подтверждается в конце выполненных действий. С простой поставленной задачей у студентов не может возникнуть сложностей, если решить уравнение онлайн в самые кратчайшие периоды времени, но не забываем о всевозможных правилах. Множество подмножеств пересекается в области сходящихся обозначений. В разных случаях произведение не ошибочно распадается на множители. Решить уравнение онлайн вам помогут в нашем первом разделе, посвященном основам математических приемов для значимых разделов для учащихся в ВУЗах и техникумах студентов. Ответные примеры нас не заставят ожидать несколько дней, так как процесс наилучшего взаимодействия векторного анализа с последовательным нахождением решений был запатентован в начале прошлого века. Выходит так, что усилия по взаимосвязям с окружающим коллективом были не напрасными, другое очевидно назрело в первую очередь. Спустя несколько поколений, ученые всего мира заставили поверить в то, что математика это царица наук. Будь-то левый ответ или правый, все равно исчерпывающие слагаемые необходимо записать в три ряда, поскольку в нашем случае речь пойдет однозначно только про векторный анализ свойств матрицы. Нелинейные и линейные уравнения, наряду с биквадратными уравнениями, заняли особый пост в нашей книге про наилучшие методы расчета траектории движения в пространстве всех материальных точек замкнутой системы. Воплотить идею в жизнь нам поможет линейный анализ скалярного произведения трех последовательных векторов. В конце каждой постановки, задача облегчается благодаря внедрениям оптимизированных числовых исключений в разрез выполняемых наложений числовых пространств. Иное суждение не противопоставит найденный ответ в произвольной форме треугольника в окружности. Угол между двумя векторами заключает в себе необходимый процент запаса и решение уравнений онлайн зачастую выявляет некий общий корень уравнения в противовес начальным условиям. Исключение выполняет роль катализатора во всем неизбежном процессе нахождения положительного решения в области определения функции. Если не сказано, что нельзя пользоваться компьютером, то калькулятор уравнений онлайн в самый раз подойдет для ваших трудных задач. Достаточно лишь вписать в правильном формате свои условные данные и наш сервер выдаст в самые кратчайшие сроки полноценный результирующий ответ. Показательная функция возрастает гораздо быстрее, чем линейная. Об этом свидетельствую талмуды умной библиотечной литературы. Произведет вычисление в общем смысле как это бы сделало данное квадратное уравнение с тремя комплексными коэффициентами. Парабола в верхней части полуплоскости характеризует прямолинейное параллельное движение вдоль осей точки. Здесь стоит упомянуть о разности потенциалов в рабочем пространстве тела. Взамен неоптимальному результату, наш калькулятор дробей по праву занимает первую позицию в математическом рейтинге обзора функциональных программ на серверной части. Легкость использования данного сервиса оценят миллионы пользователей сети интернет. Если не знаете, как им воспользоваться, то мы с радостью вам поможем. Еще хотим особо отметить и выделить кубическое уравнение из целого ряда первостепенных школьнических задач, когда необходимо быстро найти его корни и построить график функции на плоскости. Высшие степени воспроизведения — это одна из сложных математических задач в институте и на ее изучение выделяется достаточное количество часов. Как и все линейные уравнения, наши не исключение по многих объективным правилам, взгляните под разными точками зрений, и окажется просто и достаточно выставить начальные условия. Промежуток возрастания совпадает с интервалом выпуклости функции. Решение уравнений онлайн. В основе изучения теории состоят уравнения онлайн из многочисленных разделов по изучению основной дисциплины. По случаю такого подхода в неопределенных задачах, очень просто представить решение уравнений в заданном заранее виде и не только сделать выводы, но и предсказать исход такого положительного решения. Выучить предметную область поможет нам сервис в самых лучших традициях математики, именно так как это принято на Востоке. В лучшие моменты временного интервала похожие задачи множились на общий множитель в десять раз. Изобилием умножений кратных переменных в калькулятор уравнений завелось приумножать качеством, а не количественными переменными таких значений как масса или вес тела. Во избежание случаев дисбаланса материальной системы, нам вполне очевиден вывод трехмерного преобразователя на тривиальном схождении невырожденных математических матриц. Выполните задание и решите уравнение в заданных координатах, поскольку вывод заранее неизвестен, как и неизвестны все переменные, входящие в пост пространственное время. На короткий срок выдвинете общий множитель за рамки круглых скобок и поделите на наибольший общий делитель обе части заранее. Из-под получившегося накрытого подмножества чисел извлечь подробным способом подряд тридцать три точки за короткий период. Постольку поскольку в наилучшем виде решить уравнение онлайн возможно каждому студенту, забегая вперед, скажем одну важную, но ключевую вещь, без которой в дальнейшем будем непросто жить. В прошлом веке великий ученый подметил ряд закономерностей в теории математики. На практике получилось не совсем ожидаемое впечатление от событий. Однако в принципе дел это самое решение уравнений онлайн способствует улучшению понимания и восприятия целостного подхода к изучению и практическому закреплению пройдённого теоретического материала у студентов. На много проще это сделать в свое учебное время.

=

Примеры решения способом сложения | Алгебра

Рассмотрим конкретные примеры решения систем линейных уравнений способом сложения.

   

Ищем наибольший общий делитель коэффициентов при каждой из переменных (коэффициенты берем со знаком «+»).

Наименьшее общее кратное коэффициентов при x — НОК(5;2)=10, при y — НОК(3;3)=3.

Проще работать с y, поскольку для получения перед y противоположных чисел достаточно умножить любое из уравнений на -1. Проще умножить на -1 второе уравнение системы (в этом случае после сложения уравнений коэффициент при x — положительное число).

   

   

   

Теперь подставим x=3 в любое из уравнений системы, например, во второе:

   

Решаем это уравнение:

6-3y=21

-3y=21-6

-3y=15

y= -5.

   

Ответ записываем в круглых скобках через точку с запятой в алфавитном порядке.

Ответ: (3; -5).

   

НОК(6; 4)=12, НОК(13; 5)=65. Проще работать с коэффициентами перед x.

Чтобы получить перед иксами противоположные числа, первую систему умножим на -2, вторую — на 3

   

и сложим почленно левые и правые части уравнений:

   

   

Подставляем y= -1 в первое уравнение системы и находим x:

   

   

Ответ: (-2; -1).

   

НОК(3; 5)=15, НОК(5; 7)=35. Проще получить противоположные числа перед x.

Для этого умножим первое уравнение системы на 5, второе — на -3:

   

и сложим почленное левые и правые части полученных уравнений:

   

   

Подставляем y=2 в первое уравнение системы и находим x:

   

   

Ответ: (-7; 2).

   

Прежде чем применить способ сложения, данную систему следует упростить. Умножим первое уравнение на наименьший общий знаменатель дробей, во втором раскроем скобки:

   

   

   

Получили систему линейных уравнений с двумя переменными. Для решения её способом сложения достаточно умножить второе уравнение на -1 и сложить почленно левые и правые части уравнений:

   

   

   

Подставляем найденное значение b в первое уравнение системы (линейных уравнений):

   

   

Ответ: (-3; 10).

   

Систему линейных уравнений с тремя переменными можно решить, сначала исключив одно из неизвестных, а затем — другое.

В данной системе проще всего исключить переменную z.

К первому уравнению прибавим третье, умноженное на -3:

   

   

Ко второму уравнению прибавим третье, умноженное на 2:

   

   

Получили систему линейных уравнений с двумя переменными:

   

НОК(8;10)=40, НОК(13; 7)=91. Проще работать с x:

   

   

   

Подставив полученные значение y во второе уравнение системы с двумя переменными, найдём x:

   

   

Подставив значения y и x в третье уравнение системы с тремя переменными, найдём z:

   

   

Ответ: (2; 0; -1).

Решение систем уравнений: способ сложения + примеры

 

Системой линейных уравнений с двумя неизвестными — это два или несколько линейных уравнений, для которых необходимо найти все их общие решения. Мы будем рассматривать системы из двух линейных уравнений с двумя неизвестными. Общий вид системы из двух линейных уравнений с двумя неизвестными представлен на рисунке ниже:

{ a1*x + b1*y = c1,
{ a2*x + b2*y = c2

Здесь х и у неизвестные переменные, a1,a2,b1,b2,с1,с2 – некоторые вещественные числа. Решением системы двух линейных уравнений с двумя неизвестными называют пару чисел (x,y) такую, что если подставить эти числа в уравнения системы, то каждое из уравнений системы обращается в верное равенство. Существует несколько способов решения системы линейных уравнений. Рассмотрим один из способов решения системы линейных уравнений, а именно способ сложения. 

Алгоритм решения способом сложения

Алгоритм решения системы линейных уравнений с двумя неизвестными способом сложения.

1. Если требуется, путем равносильных преобразований уравнять коэффициенты при одной из неизвестных переменных в обоих уравнениях.

2. Складывая или вычитая полученные уравнения получить линейное уравнение с одним неизвестным

3. Решить полученное уравнение с одним неизвестным и найти одну из переменных.

4. Подставить полученное выражение в любое из двух уравнений системы и решить это уравнение, получив, таким образом, вторую переменную.

5. Сделать проверку решения.

Пример решения способом сложения

Для большей наглядности решим способом сложения следующую систему линейных уравнений с двумя неизвестными:

{3*x + 2*y = 10;
{5*x + 3*y = 12;

Так как, одинаковых коэффициентов нет ни у одной из переменных, уравняем коэффициенты у переменной у. Для этого умножим первое уравнение на три, а второе уравнение на два.

{3*x+2*y=10 |*3
{5*x + 3*y = 12 |*2

Получим следующую систему уравнений:

{9*x+6*y = 30;
{10*x+6*y=24;

Теперь из второго уравнения вычитаем первое. Приводим подобные слагаемые и решаем полученное линейное уравнение.

10*x+6*y – (9*x+6*y) = 24-30; x=-6;

Полученное значение подставляем в первое уравнение из нашей исходной системы и решаем получившееся уравнение.

{3*(-6) + 2*y =10;
{2*y=28; y =14;

Получилась пара чисел x=6 и y=14. Проводим проверку. Делаем подстановку.

{3*x + 2*y = 10;
{5*x + 3*y = 12;

{3*(-6) + 2*(14) = 10;
{5*(-6) + 3*(14) = 12;

{10 = 10;
{12=12;

Как видите, получились два верных равенства, следовательно, мы нашли верное решение.

Ответ: (6, 14)

Нужна помощь в учебе?



Предыдущая тема: Решение систем уравнений: способ подстановки + примеры
Следующая тема:&nbsp&nbsp&nbspРешение задач с помощью систем уравнений: общая схема решения

Примеры решения показательных уравнений

Примеры решения показательных уравнений

Примеры решения показательных уравнений

Пример №1

1000x=100

Представим левую и правую часть уравнения в виде степени, имеющую одинаковые основания:

103x=102

Теперь, когда основания одинаковые, нужно приравнять показатели степеней.

3x=2
x=2/3

Ответ: x=2/3 .

Главное в показательных уравнениях — свести левую и правую часть уравнения к общему основанию:

Пример №2

(2/5)x=(5/2)4

Представим (2/5)x как (5/2)-x:

(5/2)-x=(5/2)4

Основания одинаковые, следовательно, приравниваем показатели:

-x=4
x=-4

Ответ: x=-4

Пример №3

√3х=9

√3х распишем как 3x/2, а 9 — как 32:

3х/2=32

Приравниваем показатели:

х/2=2
х=4

Ответ: x=4

Пример №4

3х2-х-2=81

Заметим, что 81=34

3х2-х-2=34

Приравниваем показатели:

х2-х-2=4

х2-х-6=0

Получили квадратное уравнение:

D=1+24=25, D>0, следовательно, уравнение имеет два действительных корня

х1=(1+5)/2=3

х2=(1-5)/2=-2

Ответ: х=3 и х=-2

Пример №5

4х+1+4х=320

В таких случаях выносится основание с наименьшим показателем. В данном уравнении наименьшим показателем является х. Вынесем 4х за скобки:

4х(4+1)=320

4х*5=320

Представим 320 в виде 5*43, тогда:

4х*5=5*43

Поделим левую и правую часть уравнения на 5:

4х=43

Приравняем показатели:

х=3

Ответ: х=3

Пример №6

7х+2+4*7х-1=347

Степенью с наименьшим показателем в этом уравнении является х-1, следовательно, за скобки выносим 7x-1. Получаем:

7х-1*(73+4)=347

7х-1*347=347

Поделим левую и правую часть уравнения на 347:

7х-1=1

Заметим, что любое число в нулевой степени равно 1. Следовательно, распишем 1 как 70:

7х-1=70

Приравняв показатели, получим:

х-1=0

х=1

Ответ: х=1

Пример №7

4х-5*2х+4=0

Представим 4х как 2, получим:

2-5*2х+4=0

Введем подстановку: 2х обозначим переменной t. Cледовательно: 2=t2. Получим:

t2-5t+4=0

Найдем корни уравнения по теореме Виета:

t1=1

t2=4

Заменим t на 2х:

2х=1

Заметим, что 20=1

2х=20

Приравняем показатели:

х=0

2х=4

Заметим, что 4=22

2х=22

Приравняем показатели:

х=2

Уравнение имеет два действительных корня 0 и 2.

Ответ: х=0 и х=2

Пример №8

(√2+√3)х + (√2-√3)х=4

Введем подстановку: (√2+√3)х обозначим переменной t. А (√2-√3)х домножим на сопряженные и получим:

((√2+√3)х*(√2-√3)х) / (√2+√3)х = (√4-3)х/(√2+√3)х = 1 x/(2+√3)x = 1/(2+√3)x

Следовательно, 1/(√2+√3)х=1/t.

Получаем:

t+1/t=4

Отметим, что t=0, т.к. деление на 0 не определено. Домножим левую и правую часть на t:

t2+1=4t

t2-4t+1=0

Решим квадратное уравнение:

D=16-4=12, D>0, следовательно, уравнение имеет два действительных корня

t1=(4-2√3)/2=2-√3

t2=(4+2√3)/2=2+√3

Заменим t на (√2+√3)х:

(√2-√3)х=2+√3

Домножим 2+√3 на сопряженные и получим:

1/(2-√3)=2+√3

Cледовательно:

(√2-√3)х=1/2-√3

Заметим, что 1/2-√3=(√2-√3)-2

(√2+√3)х=(√2-√3)-2

Приравняв показатели, получим:

х=-2

Заменим t на 2+√3

(√2+√3)х=2+√3

Заметим, что 2+√3=(√2+√3)2

Приравняв показатели, получим:

х=2

Ответ: х=-2 и х=2

Пример №9

x+y=6

xy2+7y+12=1

Выразим x:

x=6-y

xy2+7y+12=1

Заметим, что x0=1:

x=6-y

xy2+7y+12=x0

Приравним показатели:

x=6-y

y2+7y+12=0

Решим отдельно квадратное уравнение:

y2+7y+12=0

D=49-48=1, D>0, следовательно, уравнение имеет два действительных корня

y1=(-7+1)=-3

y2=(-7-1)=-4

y=-3

x=6-(-3)=9

y=-4

x=6-(-4)=10

Ответ: x=9; y=-3 и x=10; y=-4

<< Назад ] [ Начало ] [ Вперед >>


определение, виды, примеры решения, что это такое

Статья знакомит с таким понятием, как определение системы уравнений и ее решением. Будут рассмотрены часто встречающиеся случаи решений систем. Приведенные примеры помогут подробно пояснить решение.

Определение системы уравнений

Чтобы перейти к определению системы уравнений, необходимо обратить внимание на два момента: вид записи и ее смысл. Чтобы понять это, нужно подробно остановиться на каждом из видов, тогда сможем прийти к определению систем уравнений.

Например, возьмем два уравнения 2·x+y=−3 и x=5, после чего объединим фигурной скобкой такого плана:

2·x+y=-3,x=5.

Уравнения, объединенные фигурной скобкой, считаются записями систем уравнений. Они задают множества решений уравнений данной системы. Каждое решение должно являться решением всех заданных уравнений.

Другими словами это означает, что любые решения первого уравнения будут решениями всех уравнений, объединенных системой.

Определение 1

Системы уравнений – это некоторое количество уравнений, объединенных фигурной скобкой, имеющих множество решений уравнений, которые одновременно являются решениями для всей системы.

Основные виды систем уравнений

Видов уравнений достаточно много, как систем уравнений. Для того, чтобы было удобно решать и изучать их, подразделяют на группы по определенным характеристикам. Это поможет в рассмотрении систем уравнений отдельных видов.

Для начала уравнения классифицируются по количеству уравнений. Если уравнение одно, то оно является обычным уравнением, если их более, тогда имеем дело с системой, состоящей из двух или более уравнений.

Другая классификация затрагивает число переменных. Когда количество переменных 1, говорят, что имеем дело с системой уравнений с одной неизвестной, когда 2 – с двумя переменными. Рассмотрим пример

x+y=5,2·x-3·y=1

Очевидно, что система уравнений включает в себя две переменные х и у.

При записи таких уравнений считается число всех переменных, имеющихся в записи. Их наличие в каждом уравнении необязательно. Хотя бы одно уравнение должно иметь одну переменную. Рассмотрим пример системы уравнений

2x=11,x-3·z2=0,27·x+y-z=-3

Данная система имеет 3 переменные х, у, z. Первое уравнение имеет явный х и неявные у и z. Неявные переменные – это переменные, имеющие 0 в коэффициенте. Второе уравнение имеет х и z, а у неявная переменная. Иначе это можно записать таким образом

2x+0·y+0·z=11

А другое уравнение x+0·y−3·z=0.

Третья классификация уравнений – это вид. В школе проходят простые уравнения и системы уравнений, начиная с систем двух линейных уравнений с двумя переменными. Имеется в виду, что система включает в себя 2 линейных уравнения. Для примера рассмотрим

2·x-y=1,x+2·y=-1и -3·x+y=0.5,x+223·y=0

Это основные простейшие линейные уравнения. Далее можно столкнуться с системами, содержащими 3 и более неизвестных.

В 9 классе решают уравнения с двумя переменными и нелинейные. В целых уравнениях повышается степень для увеличения сложности. Такие системы называют системами нелинейных уравнений с определенным количеством уравнений и неизвестных. Рассмотрим примеры таких систем

x2-4·x·y=1,x-y=2 и x=y3x·y=-5

Обе системы с двумя переменными и обе являются нелинейными.

При решении можно встретить дробно-рациональные уравнения. Например

x+y=3,1x+1y=25

Могут называть просто системой уравнений без уточнения, каких именно. Редко уточняют сам вид системы.

Нужна помощь преподавателя?

Опиши задание — и наши эксперты тебе помогут!

Описать задание

Старшие классы переходят к изучению иррациональных, тригонометрических и показательных уравнений. Например,

x+y-x·y=5,2·x·y=3, x+y=5·π2,sin x+cos 2y=-1,y-log3x=1,xy=312.

Высшие учебные заведения изучают и исследуют решения систем линейных алгебраических уравнений (СЛАУ). Левая часть таких уравнений содержит многочлены с первой степенью, а правая – некоторые числа. Отличие от школьных в том, что количество переменных и количество уравнений может быть произвольным, чаще всего несовпадающим.

Решение систем уравнений

Определение 2

Решение системы уравнений с двумя переменными – это пара переменных, которая при подстановке обращает каждое уравнение в верное числовое неравенство, то есть является решением для каждого уравнения данной системы.

К примеру, пара значений х=5 и у=2 являются решением системы уравнений x+y=7,x-y=3. Потому как при подстановке уравнения обращаются в верные числовые неравенства 5+2=7 и 5−2=3. Если подставить пару х=3 и у=0, тогда система не будет решена, так как подстановка не даст верное уравнение, а именно, мы получим 3+0=7.

Сформулируем определение для систем, содержащих одну и более переменных.

Определение 3

Решение системы уравнений с одной переменной – это значение переменной, которая является корнем уравнений системы, значит, все уравнения будут обращены в верные числовые равенства.

Рассмотрим на примере системы уравнений с одной переменной t

t2=4,5·(t+2)=0

Число -2 – решение уравнения, так как  (−2)·2=4, и 5·(−2+2)=0 являются верными числовыми равенствами. При t=1 система не решена, так как при подстановке получим два неверных равенства 12=4 и 5·(1+2)=0.

Определение 4

Решение системы с тремя и более переменными называют тройку, четверку и далее значений соответственно, которые обращают все уравнения системы в верные равенства.

Если имеем значения переменных х=1, у=2, z=0, то подставив их в систему уравнений 2·x=2,5·y=10,x+y+z=3, получим 2·1=2, 5·2=10 и 1+2+0=3. Значит, эти числовые неравенства верные. А значения (1, 0, 5) не будут решением, так как, подставив значения, второе из них будет неверное, как и третье: 5·0=10, 1+0+5=3.

Системы уравнений могут не иметь решений вовсе или иметь бесконечное множество. В этом можно убедиться при углубленном изучении данной тематики. Можно прийти к выводу, что системы уравнений – это пересечение множеств решений всех ее уравнений. Раскроем несколько определений:

Определение 5

Несовместной называют систему уравнений, когда она не имеет решений, в противном случае ее называют совместной.

Определение 6

Неопределенной называют систему, когда она имеет бесконечное множество решений, а определенной при конечном числе решений либо при их отсутствии.

Такие термины редко применяются в школе, так как рассчитаны для программ высших учебных заведений. Знакомство с равносильными системами углубит имеющиеся знания по решению систем уравнений.

Решение уравнений

Решение уравнений с одной переменной

An уравнение представляет собой математическое выражение, состоящее из знака равенства между двумя числовыми выражениями или выражениями переменных, как в 3 Икс + 5 знак равно 11 .

А решение к уравнению это число который может быть подключен к Переменная сделать истинное числовое утверждение.

Пример 1:

Подстановка 2 для Икс в

3 Икс + 5 знак равно 11

дает

3 ( 2 ) + 5 знак равно 11 , в котором говорится 6 + 5 знак равно 11 ; это правда!

Так 2 это решение.

По факту, 2 ЕДИНСТВЕННОЕ решение 3 Икс + 5 знак равно 11 .

Некоторые уравнения могут иметь более одного решения, бесконечно много решений или вообще не иметь решений.

Пример 2:

Уравнение

Икс 2 знак равно Икс

имеет два решения, 0 и 1 , поскольку

0 2 знак равно 0 и 1 2 знак равно 1 .Никакой другой номер не работает.

Пример 3:

Уравнение

Икс + 1 знак равно 1 + Икс

верно для все реальные числа . Она имеет бесконечно много решения.

Пример 4:

Уравнение

Икс + 1 знак равно Икс

является никогда верно для любой настоящий номер.Она имеет нет решений .

В задавать содержащее все решения уравнения, называется набор решений для этого уравнения.

Уравнение

Набор решений

3 Икс + 5 знак равно 11

{ 2 }

Икс 2 знак равно Икс

{ 0 , 1 }

Икс + 1 знак равно 1 + Икс

р (набор всех действительных чисел)

Икс + 1 знак равно Икс

(пустой набор)

Иногда вас могут попросить решить уравнение над определенным домен .Здесь возможности для значений Икс ограничены.

Пример 5:

Решите уравнение

Икс 2 знак равно Икс

по домену { 0 , 1 , 2 , 3 } .

Это немного сложное уравнение; это не линейный и это не квадратичный , поэтому у нас нет хорошего метода ее решения.Однако, поскольку домен содержит только четыре числа, мы можем просто использовать метод проб и ошибок.

0 2 знак равно 0 знак равно 0 1 2 знак равно 1 знак равно 1 2 2 ≠ 2 3 2 ≠ 3

Так что набор решений в данном домене { 0 , 1 } .

Решение уравнений с двумя переменными

Решения для уравнения с одной переменной: числа . С другой стороны, решения уравнения с двумя переменными имеют вид заказанные пары в виде ( а , б ) .

Пример 6:

Уравнение

Икс знак равно у + 1

верно, когда Икс знак равно 3 и у знак равно 2 .Итак, заказанная пара

( 3 , 2 )

является решением уравнения.

Есть бесконечно много других решений этого уравнения, например:

( 4 , 3 ) , ( 11 , 10 ) , ( 5.5 , 4.5 ) , и т.п.

Упорядоченные пары, которые являются решениями уравнения с двумя переменными, можно изобразить на декартова плоскость . Результатом может быть линия или интересная кривая, в зависимости от уравнения. Смотрите также построение графиков линейных уравнений и построение графиков квадратных уравнений .

Решение линейных уравнений | Уравнения и неравенства

Упражнение 4.1

\ begin {align *} 2г — 3 & = 7 \\ 2л & = 10 \\ y & = 5 \ end {выровнять *}

\ begin {align *} 2c & = c — 8 \\ c & = -8 \ end {выровнять *}

\ begin {align *} 3 & = 1 — 2c \\ 2c & = 1 — (3) \\ 2c & = -2 \\ c & = \ frac {-2} {2} \\ & = -1 \ end {align *}

\ begin {align *} 4b +5 & = -7 \\ 4b & = -7 — (5) \\ 4b & = -12 \\ b & = \ frac {-12} {4} \\ & = -3 \ end {align *}

\ begin {align *} -3y & = 0 \\ у & = 0 \ end {выровнять *}

\ begin {align *} 16л + ​​4 & = -10 \\ 16лет & = -14 \\ y & = — \ frac {14} {16} \\ & = — \ frac {7} {8} \ end {выровнять *}

\ begin {align *} 12лет + 0 & = 144 \\ 12лет & = 144 \\ y & = 12 \ end {выровнять *}

\ begin {align *} 7 + 5л & = 62 \\ 5лет & = 55 \\ y & = 11 \ end {выровнять *}

\ (55 = 5x + \ frac {3} {4} \)

\ begin {align *} 55 & = 5x + \ frac {3} {4} \\ 220 & = 20х + 3 \\ 20x & = 217 \\ х & = \ frac {217} {20} \ end {выровнять *}

\ begin {align *} 5х & = 2х + 45 \\ 3x & = 45 \\ х & = 15 \ end {выровнять *}

\ begin {align *} 23х — 12 & = 6 + 3х \\ 20x & = 18 \\ x & = \ frac {18} {20} \\ & = \ frac {9} {10} \ end {выровнять *}

\ (12 — 6x + 34x = 2x — 24 — 64 \)

\ begin {align *} 12 — 6x + 34x & = 2x — 24 — 64 \\ 12 + 28x & = 2x — 88 \\ 26x & = -100 \\ x & = — \ frac {100} {26} \\ & = — \ frac {50} {13} \ end {выровнять *}

\ (6x + 3x = 4-5 (2x — 3) \)

\ begin {align *} 6x + 3x & = 4-5 (2x — 3) \\ 9x & = 4 — 10x + 15 \\ 19x & = 19 \\ х & = 1 \ end {выровнять *}

\ begin {align *} 18 — 2р & = р + 9 \\ 9 & = 3п \\ p & = 3 \ end {выровнять *}

\ (\ dfrac {4} {p} = \ dfrac {16} {24} \)

\ begin {align *} \ frac {4} {p} & = \ frac {16} {24} \\ (4) (24) & = (16) (p) \\ 16p & = 96 \\ p & = 6 \ end {выровнять *}

\ begin {align *} — (- 16 — п) & = 13п — 1 \\ 16 + п & = 13п — 1 \\ 17 & = 12п \\ p & = \ frac {17} {12} \ end {выровнять *}

\ begin {align *} 3f — 10 & = 10 \\ 3f & = 20 \\ f & = \ frac {20} {3} \ end {выровнять *}

\ begin {align *} 3f + 16 & = 4f — 10 \\ f & = 26 \ end {выровнять *}

\ (10f + 5 = -2f -3f + 80 \)

\ begin {align *} 10f + 5 & = -2f — 3f + 80 \\ 10f + 5 & = -5f + 80 \\ 15f & = 75 \\ f & = 5 \ end {выровнять *}

\ begin {align *} 8 (ф — 4) & = 5 (ф — 4) \\ 8f — 32 & = 5f — 20 \\ 3f & = 12 \\ f & = 4 \ end {выровнять *}

\ begin {align *} 6 & = 6 (f + 7) + 5f \\ 6 & = 6f + 42 + 5f \\ -36 & = 11f \\ f & = — \ frac {36} {11} \ end {выровнять *}

\ begin {align *} -7x & = 8 (1 — х) \\ -7x & = 8 — 8x \\ х & = 8 \ end {выровнять *}

\ (5 — \ dfrac {7} {b} = \ dfrac {2 (b + 4)} {b} \)

\ begin {align *} 5 — \ frac {7} {b} & = \ frac {2 (b + 4)} {b} \\ \ frac {5b — 7} {b} & = \ frac {2b + 8} {b} \\ 5b — 7 & = 2b + 8 \\ 3b & = 15 \\ b & = 5 \ end {выровнять *}

\ (\ dfrac {x + 2} {4} — \ dfrac {x — 6} {3} = \ dfrac {1} {2} \)

\ begin {align *} \ frac {x + 2} {4} — \ frac {x — 6} {3} & = \ frac {1} {2} \\ \ frac {3 (x + 2) — 4 (x — 6)} {12} & = \ frac {1} {2} \\ \ frac {3x + 6 — 4x + 24} {12} & = \ frac {1} {2} \\ (-x + 30) (2) & = 12 \\ -2x + 60 & = 12 \\ -2x & = -48 \\ х & = 24 \ end {выровнять *}

\ (1 = \ dfrac {3a — 4} {2a + 6} \)

Обратите внимание, что \ (a \ neq — -3 \)

\ begin {align *} 1 & = \ frac {3a — 4} {2a + 6} \\ 2а + 6 & = 3а — 4 \\ а & = 10 \ end {выровнять *}

\ (\ dfrac {2-5a} {3} — 6 = \ dfrac {4a} {3} +2 — a \)

\ begin {align *} \ frac {2-5a} {3} — 6 & = \ frac {4a} {3} +2 — a \\ \ frac {2-5a} {3} — \ frac {4a} {3} + a & = 8 \\ \ frac {2-5a — 4 a + 3a} {3} & = 8 \\ 2 — 6а & = 24 \\ 6а & = -22 \\ a & = — \ frac {22} {6} \ end {выровнять *}

\ (2 — \ dfrac {4} {b + 5} = \ dfrac {3b} {b + 5} \)

Примечание \ (b \ neq -5 \)

\ begin {align *} 2 — \ frac {4} {b + 5} & = \ frac {3b} {b + 5} \\ 2 & = \ frac {3b + 4} {b + 5} \\ 2b + 10 & = 3b + 4 \\ b & = 6 \ end {выровнять *}

\ (3 — \ dfrac {y — 2} {4} = 4 \)

\ begin {align *} 3 — \ frac {y — 2} {4} & = 4 \\ — \ frac {y — 2} {4} & = 1 \\ -у + 2 & = 4 \\ y & = -2 \ end {выровнять *}

\ (\ text {1,5} x + \ text {3,125} = \ text {1,25} x \)

\ begin {align *} \ text {1,5} x + \ text {3,125} & = \ text {1,25} x \\ \ text {1,5} x — \ text {1,25} x & = — \ text {3,125} \\ \ text {0,25} x & = — \ text {3,125} \\ х & = — \ текст {12,5} \ end {выровнять *}

\ (\ текст {1,3} (\ текст {2,7} х + 1) = \ текст {4,1} — х \)

\ begin {align *} \ text {1,3} (\ text {2,7} x + 1) & = \ text {4,1} — x \\ \ text {3,51} x + \ text {1,3} & = \ text {4,1} — x \\ \ text {4,51} x & = \ text {2,8} \\ x & = \ frac {\ text {2,8}} {\ text {4,51}} \\ & = \ frac {280} {451} \ end {выровнять *}

\ (\ текст {6,5} х — \ текст {4,15} = 7 + \ текст {4,25} х \)

\ begin {align *} \ text {6,5} x — \ text {4,15} & = 7 + \ text {4,25} x \\ \ text {2,25} x & = \ text {11,15} \\ x & = \ frac {\ text {11,15}} {\ text {2,25}} \\ & = \ frac {\ text {1 115}} {225} \\ & = \ frac {223} {45} \ end {выровнять *}

\ (\ frac {1} {3} P + \ frac {1} {2} P — 10 = 0 \)

\ begin {align *} \ frac {1} {3} P + \ frac {1} {2} P — 10 & = 0 \\ \ frac {2 + 3} {6} P & = 10 \\ 5П & = 60 \\ P & = 12 \ end {выровнять *}

\ (1 \ frac {1} {4} (x — 1) — 1 \ frac {1} {2} (3x + 2) = 0 \)

\ begin {align *} 1 \ frac {1} {4} (x — 1) — 1 \ frac {1} {2} (3x + 2) & = 0 \\ \ frac {5} {4} x — \ frac {5} {4} — \ frac {3} {2} (3x) — \ frac {3} {2} (2) & = 0 \\ \ frac {5} {4} x — \ frac {5} {4} — \ frac {9} {2} x — \ frac {6} {2} & = 0 \\ \ frac {5 — 18} {4} x + \ frac {-5 — 12} {4} & = 0 \\ \ frac {-13} {4} x & = \ frac {17} {4} \\ -13x & = 17 \\ х & = — \ frac {17} {13} \ end {выровнять *}

\ (\ frac {1} {5} (x- 1) = \ frac {1} {3} (x-2) + 3 \)

\ begin {align *} \ frac {1} {5} (x- 1) & = \ frac {1} {3} (x-2) + 3 \\ \ frac {1} {5} x- \ frac {1} {5} & = \ frac {1} {3} x- \ frac {2} {3} + 3 \\ — \ frac {1} {5} + \ frac {2} {3} — 3 & = \ frac {2} {15} x \\ — \ frac {38} {15} & = \ frac {2} {15} x \\ х & = — \ frac {38} {2} \\ х & = -19 \ end {выровнять *}

\ (\ dfrac {5} {2a} + \ dfrac {1} {6a} — \ dfrac {3} {a} = 2 \)

\ begin {align *} \ frac {5} {2a} + \ frac {1} {6a} — \ frac {3} {a} & = 2 \\ \ frac {5 (3) + 1-3 (6)} {6a} & = 2 \\ \ frac {15 + 1 — 18} {6a} & = 2 \\ \ frac {-2} {6a} & = 2 \\ -2 & = 12а \\ а & = — \ frac {1} {6} \ end {выровнять *}

Решение линейных уравнений с одной переменной

Линейные уравнения с одной переменной — это уравнения, в которых переменная имеет показатель степени 1, который обычно не отображается (понимается).Примером может быть что-то вроде \ (12x = x — 5 \). Для решения линейных уравнений есть одна основная цель: изолировать переменную . В этом уроке мы рассмотрим, как это делается, на нескольких примерах.

Содержание

  1. Примеры решения одношаговых уравнений
  2. Примеры решения двухэтапных уравнений
  3. Примеры уравнений, в которых сначала необходимо упростить
  4. Бесконечно много или нет решений
  5. Сводка

реклама

Примеры решения одношаговых линейных уравнений

После всей вашей тяжелой работы над решением уравнения вы знаете, что хотите получить окончательный ответ, например \ (x = 5 \) или \ (y = 1 \).В обоих случаях переменная изолирована, или сама по себе.

Итак, нам нужно выяснить, как изолировать переменную. Как мы это сделаем, зависит от самого уравнения! Если его на что-то умножили, поделим. Если к нему что-то добавили, мы вычтем. Поступая так, мы постепенно будем получать переменную сама по себе.

Давайте рассмотрим пример, чтобы увидеть, как это работает.

Пример

Решите уравнение: \ (4x = 8 \)

Решение

В этом примере 4 — это умножение на \ (x \).Следовательно, чтобы изолировать \ (x \), вы должны разделить эту сторону на 4. Делая это, вы должны помнить одно важное правило: что бы вы ни делали с одной стороной уравнения, вы должны делать с другой стороной. Итак, мы разделим обе стороны на 4.

\ (\ begin {align} 4x & = 8 \\ \ dfrac {4x} {\ color {red} {4}} & = \ dfrac {8} {\ color {red} {4}} \ end {align} \)

Упрощение:

\ (х = \ в коробке {2} \)

Вот и все, один шаг и готово. (Вот почему подобные уравнения часто называют «одношаговыми» уравнениями)

Чек

Каждый раз, когда вы решаете линейные уравнения, вы всегда можете проверить свой ответ, подставив его обратно в уравнение.Если вы получите верное утверждение, значит, ответ правильный. Это не обязательно на 100% для каждой задачи, но это хорошая привычка, поэтому мы сделаем это для наших уравнений.

В этом примере наше исходное уравнение было \ (4x = 8 \). Чтобы проверить это, убедитесь, что верно следующее:

\ (\ begin {align} 4x & = 8 \\ 4 (2) & = 8 \\ 8 & = 8 \ end {align} \)

Это верное утверждение, поэтому наш ответ правильный.

Для любого уравнения любая операция, которую вы выполняете с одной стороной, должна выполняться и с другой стороной.

Давайте попробуем еще пару примеров, прежде чем переходить к более сложным уравнениям.

Пример

Решить: \ (3x = 12 \)

Решение

Поскольку \ (x \) умножается на 3, план состоит в том, чтобы разделить на 3 с обеих сторон:

\ (\ begin {align} 3x & = 12 \\ \ dfrac {3x} {\ color {red} {3}} & = \ dfrac {12} {\ color {red} {3}} \\ x & = \ в штучной упаковке {4} \ end {align} \)

Чек

Чтобы проверить наш ответ, мы позволим \ (x = 4 \) и подставим его обратно в уравнение:

\ (\ begin {align} 3x & = 12 \\ 3 (4) & = 12 \\ 12 & = 12 \ end {align} \)

Как и раньше, поскольку это верное утверждение, мы знаем, что наш ответ правильный.

В следующем примере вместо умножения переменной на значение из переменной вычитается значение. Чтобы «отменить» это, мы добавим это значение обеим сторонам.

Пример

Решить: \ (y-9 = 21 \)

Решение

На этот раз из y вычитается 9. Итак, мы отменим это, добавив 9 к обеим сторонам.

\ (\ begin {align} y-9 & = 21 \\ y-9 \ color {red} {+ 9} & = 21 \ color {red} {+ 9} \\ y & = 30 \ end {align} \)

Далее мы рассмотрим то, что обычно называют «двухэтапными» уравнениями.В этих уравнениях нам нужно будет отменить две операции, чтобы изолировать переменную.

Примеры двухступенчатых уравнений

В каждом из приведенных выше примеров нужно было выполнить один шаг, прежде чем мы получили ответ. В следующих примерах вы увидите, как работать с уравнениями, которые вместо этого состоят из двух шагов. Если выполняется более одной операции, важно помнить порядок операций PEMDAS. Поскольку вы отменяете операции с \ (x \), вы будете работать «снаружи внутрь».Это легче понять, когда вы увидите это на примере.

Пример

Решить: \ (2x-7 = 13 \)

Решение

Обратите внимание на две операции, выполняемые с \ (x \): он умножается на 2, а затем вычитается 7. Нам нужно будет их отменить. Но только \ (x \) умножается на 2, поэтому первым шагом будет прибавление 7 к обеим сторонам. Тогда мы можем разделить обе части на 2.

Добавляем 7 к обеим сторонам:

\ (\ begin {align} 2x-7 & = 13 \\ 2x-7 \ color {red} {+ 7} & = 13 \ color {red} {+ 7} \\ 2x & = 20 \ end {align} \ )

Теперь разделите обе стороны на 2:

.

\ (\ begin {align} 2x & = 20 \\ \ dfrac {2x} {\ color {red} {2}} & = \ dfrac {20} {\ color {red} {2}} \\ x & = \ в штучной упаковке {10} \ end {align} \)

Чек

Как и в случае с более простыми задачами, вы можете проверить свой ответ, подставив свое значение \ (x \) обратно в исходное уравнение.

\ (\ begin {align} 2x-7 & = 13 \\ 2 (10) — 7 & = 13 \\ 13 & = 13 \ end {align} \)

Это правда, значит, у нас есть правильный ответ.

Давайте рассмотрим еще один пример с двумя шагами, прежде чем мы снова будем преодолевать трудности. Убедитесь, что вы понимаете каждый показанный шаг и также работаете над проблемой.

Пример

Решить: \ (5w + 2 = 9 \)

Решение

Как и выше, есть две операции: \ (w \) умножается на 5, а затем к нему прибавляется 2.Мы отменим их, сначала вычтя 2 с обеих сторон, а затем разделив на 5.

\ (\ begin {align} 5w + 2 & = 9 \\ 5w + 2 \ color {red} {- 2} & = 9 \ color {red} {- 2} \\ 5w & = 7 \\ \ dfrac { 5w} {\ color {red} {5}} & = \ dfrac {7} {\ color {red} {5}} \\ w = \ boxed {\ dfrac {7} {5}} \ end {align} \)

Дробь справа не может быть упрощена, так что это наш окончательный ответ.

Чек

Пусть \ (w = \ dfrac {7} {5} \). Тогда:

\ (\ begin {align} 5w + 2 & = 9 \\ 5 \ left (\ dfrac {7} {5} \ right) + 2 & = 9 \\ 7 + 2 & = 9 \\ 9 & = 9 \ конец {align} \)

Итак, мы снова получили правильный ответ!

Упрощение перед решением

В следующих примерах есть больше вариативных терминов и, возможно, необходимо некоторое упрощение.В каждом случае шаги будут заключаться в том, чтобы сначала упростить обе стороны, а затем использовать то, что мы делали, чтобы изолировать переменную. Сначала мы подробно рассмотрим пример, чтобы увидеть, как все это работает.

Чтобы понять этот раздел, вам должно быть удобно комбинировать похожие термины.

Пример

Решить: \ (3x + 2 = 4x-1 \)

Решение

Поскольку обе части упрощены (нет скобок, которые нам нужно вычислять, и нет одинаковых членов для объединения), следующим шагом будет получение всех x на одной стороне уравнения и всех чисел на другой стороне.Применяется то же правило — что бы вы ни делали с одной стороной уравнения, вы должны делать и с другой стороной!

Можно перемещать \ (3x \) или \ (4x \). Предположим, вы переместили \ (4x \). Поскольку он положительный, вы должны вычесть его с обеих сторон:

\ (\ begin {align} 3x + 2 & = 4x-1 \\ 3x + 2 \ color {red} {- 4x} & = 4x-1 \ color {red} {- 4x} \\ -x + 2 & = -1 \ end {align} \)

Теперь уравнение выглядит так же, как и раньше. Следующий шаг — вычесть 2 с обеих сторон:

\ (\ begin {align} -x + 2 \ color {red} {- 2} & = -1 \ color {red} {- 2} \\ — x = -3 \ end {align} \)

Наконец, поскольку \ (- x = -1x \) (это всегда верно), разделите обе стороны на \ (- 1 \):

\ (\ begin {align} \ dfrac {-x} {\ color {red} {- 1}} & = \ dfrac {-3} {\ color {red} {- 1}} \\ x & = 3 \ end {выровнять}\)

Чек

Вы должны воспользоваться моментом и убедиться, что следующее утверждение является верным:

\ (3 (3) + 2 = 4 (3) — 1 \)

В следующем примере нам нужно будет использовать свойство распределения перед решением.Здесь легко ошибиться, поэтому убедитесь, что вы распределили число перед круглыми скобками для всех терминов внутри.

Пример

Решить: \ (3 (x + 2) -1 = x-3 (x + 1) \)

Решение

Сначала разложите 3 и –3 и соберите одинаковые термины.

\ (\ begin {align} 3 (x + 2) -1 & = x-3 (x + 1) \\ 3x + 6-1 & = x-3x-3 \\ 3x + 5 & = — 2x-3 \ end {выровнять}\)

Теперь мы можем прибавить 2x к обеим сторонам. (Помните, что вы получите тот же ответ, если вместо этого вычтете 3x с обеих сторон)

\ (\ begin {align} 3x + 5 \ color {red} {+ 2x} & = — 2x-3 \ color {red} {+ 2x} \\ 5x + 5 & = -3 \ end {align} \)

Отсюда мы можем решить, как и с другими двухшаговыми уравнениями.

\ (\ begin {align} 5x + 5 \ color {red} {- 5} & = — 3 \ color {red} {- 5} \\ 5x & = — 8 \\ \ dfrac {5x} {\ color { красный} {5}} & = \ dfrac {-8} {\ color {red} {5}} \\ x & = \ dfrac {-8} {5} \\ & = \ boxed {- \ dfrac {8 } {5}} \ end {align} \)

Чек

Это был сложный вопрос, поэтому не забудьте проверить свой ответ и убедиться, что не было допущено никаких ошибок. Для этого вы убедитесь, что следующее утверждение является верным:

\ (3 \ left (- \ dfrac {8} {5} +2 \ right) -1 = \ left (- \ dfrac {8} {5} \ right) -3 \ left (- \ dfrac {8} { 5} +1 \ вправо) \)

(Примечание: это работает, но вы должны быть очень осторожны с скобками!)

Бесконечно много решений и нет решений

Бывают случаи, когда вы выполняете все эти шаги, и появляется действительно странное решение.Например, при решении уравнения \ (x + 2 = x + 2 \) с использованием описанных выше шагов в итоге получается \ (0 = 0 \). Это, конечно, правда, но что хорошего в этом?

Если вы получили подобное утверждение, это означает, что уравнение имеет бесконечно много решений. Любой \ (x \), о котором вы можете подумать, удовлетворял бы уравнению \ (x + 2 = x + 2 \). Подходящий ответ в этом случае — «бесконечно много решений».

Другая ситуация возникает, когда вы упрощаете уравнение до утверждения, которое никогда не бывает истинным, например \ (3 = 4 \) или \ (0 = 1 \).Это происходит с уравнением \ (x + 5 = x-7 \), которое приводит к \ (5 = -7 \), что, конечно, никогда не бывает истинным. Это означает, что никакое \ (x \) не удовлетворяет этому уравнению. Другими словами «решения нет». Итого:

  • Если вы получите утверждение, которое всегда истинно, например \ (5 = 5 \) или \ (0 = 0 \), то существует бесконечно много решений.
  • Если вы получаете утверждение, которое всегда ложно, например \ (10 ​​= 11 \) или \ (1 = 5 \), то решений нет.

реклама

Сводка

Решение линейных уравнений сводится к выделению переменной.В зависимости от уравнения это может занять от одного шага до многих. Всегда проверяйте, нужно ли вам сначала упростить одну или обе стороны уравнения, и всегда проверяйте свой ответ.

Подпишитесь на нашу рассылку новостей!

Мы всегда публикуем новые бесплатные уроки и добавляем новые учебные пособия, руководства по калькуляторам и пакеты задач.

Подпишитесь, чтобы получать электронные письма (раз в пару или три недели) с информацией о новинках!

Связанные

Решение линейных уравнений с одной переменной

Линейное уравнение — это уравнение прямой, записанное с одной переменной.Единственная степень переменной — 1. Линейные уравнения с одной переменной могут иметь вид [latex] ax + b = 0 [/ latex] и решаются с использованием основных алгебраических операций.

Мы начинаем с классификации линейных уравнений с одной переменной как одного из трех типов: тождественные, условные или противоречивые. Уравнение идентичности верно для всех значений переменной. Вот пример тождественного уравнения.

[латекс] 3x = 2x + x [/ латекс]

Набор решений состоит из всех значений, которые делают уравнение истинным.Для этого уравнения набором решений является все действительные числа, потому что любое действительное число, замененное на [латекс] x [/ латекс], сделает уравнение истинным.

Условное уравнение верно только для некоторых значений переменной. Например, если нам нужно решить уравнение [латекс] 5x + 2 = 3x — 6 [/ latex], мы получим следующее:

[латекс] \ begin {array} {l} 5x + 2 \ hfill & = 3x — 6 \ hfill \\ 2x \ hfill & = — 8 \ hfill \\ x \ hfill & = — 4 \ hfill \ end {array} [/ латекс]

Набор решений состоит из одного числа: [латекс] \ {- 4 \} [/ латекс].Это единственное решение, поэтому мы решили условное уравнение.

Непоследовательное уравнение приводит к ложному утверждению. Например, если мы должны решить [латекс] 5x — 15 = 5 \ left (x — 4 \ right) [/ latex], мы получим следующее:

[латекс] \ begin {array} {ll} 5x — 15 = 5x — 20 \ hfill & \ hfill \\ 5x — 15 — 5x = 5x — 20 — 5x \ hfill & \ text {Вычесть} 5x \ text {из обе стороны}. \ hfill \\ -15 \ ne -20 \ hfill & \ text {Ложный оператор} \ hfill \ end {array} [/ latex]

Действительно, [латекс] -15 \ ne -20 [/ латекс].Нет решения, потому что это противоречивое уравнение.

Решение линейных уравнений с одной переменной включает фундаментальные свойства равенства и основные алгебраические операции. Ниже приводится краткий обзор этих операций.

Общее примечание: линейное уравнение с одной переменной

Линейное уравнение с одной переменной можно записать в виде

[латекс] ax + b = 0 [/ латекс]

, где a и b — действительные числа, [латекс] a \ ne 0 [/ латекс].

Как сделать: дано линейное уравнение с одной переменной, используйте алгебру для его решения.

Следующие шаги используются для манипулирования уравнением и выделения неизвестной переменной, так что последняя строка читается как x = _________, если x — неизвестное. Нет установленного порядка, так как используемые шаги зависят от того, что указано:

  1. Мы можем складывать, вычитать, умножать или делить уравнение на число или выражение, если мы делаем то же самое с обеими сторонами знака равенства.Обратите внимание, что мы не можем делить на ноль.
  2. Примените свойство распределения по мере необходимости: [latex] a \ left (b + c \ right) = ab + ac [/ latex].
  3. Выделите переменную на одной стороне уравнения.
  4. Когда переменная умножается на коэффициент на последнем этапе, умножьте обе части уравнения на обратную величину коэффициента.

Пример 1: Решение уравнения с одной переменной

Решите следующее уравнение: [латекс] 2x + 7 = 19 [/ латекс].

Решение

Это уравнение может быть записано в виде [латекс] ax + b = 0 [/ латекс] путем вычитания [латекс] 19 [/ латекс] с обеих сторон.Однако мы можем перейти к решению уравнения в его исходной форме, выполнив алгебраические операции.

[латекс] \ begin {array} {ll} 2x + 7 = 19 \ hfill & \ hfill \\ 2x = 12 \ hfill & \ text {Вычтите 7 с обеих сторон}. \ Hfill \\ x = 6 \ hfill & \ text {Умножьте обе стороны на} \ frac {1} {2} \ text {или разделите на 2}. \ hfill \ end {array} [/ latex]

Решение [латекс] x = 6 [/ латекс].

Попробуй 1

Решите линейное уравнение с одной переменной: [латекс] 2x + 1 = -9 [/ латекс].

Решение

Пример 2: Алгебраическое решение уравнения, когда переменная появляется с обеих сторон

Решите следующее уравнение: [латекс] 4 \ left (x — 3 \ right) + 12 = 15-5 \ left (x + 6 \ right) [/ latex].

Решение

Примените стандартные алгебраические свойства.

[латекс] \ begin {array} {ll} 4 \ left (x — 3 \ right) + 12 = 15-5 \ left (x + 6 \ right) \ hfill & \ hfill \\ 4x — 12 + 12 = 15 — 5x — 30 \ hfill & \ text {Применить свойство распределения}. \ Hfill \\ 4x = -15 — 5x \ hfill & \ text {Объединить похожие термины}. \ Hfill \\ 9x = -15 \ hfill & \ text {Поместите} x- \ text {термины на одну сторону и упростите}. \ hfill \\ x = — \ frac {15} {9} \ hfill & \ text {Умножьте обе стороны на} \ frac {1} {9 } \ text {, обратное 9}.\ hfill \\ x = — \ frac {5} {3} \ hfill & \ hfill \ end {array} [/ latex]

Анализ решения

Эта задача требует, чтобы свойство распределения применялось дважды, а затем свойства алгебры используются для достижения последней строки, [latex] x = — \ frac {5} {3} [/ latex].

Попробуй 2

Решите уравнение с одной переменной: [латекс] -2 \ left (3x — 1 \ right) + x = 14-x [/ latex].

Решение

Решение линейных уравнений с нулевым Солнцем, Без Солнца и «Все-x» Солнцем

Purplemath

Есть три типа решений, которые могут вызвать путаницу.Мы рассмотрим по одному примеру каждого из них, и я объясню различия. Затем мы поработаем над смесью типов уравнений, чтобы вам было удобнее различать типы решений.

Чтобы решить это уравнение, мне сначала нужно упростить левую часть, взяв «минус» в скобки и объединив «похожие» термины:

MathHelp.com

5 — (3 х + 4)

5 — 1 (3 x ) — 1 (+4)

5–3 x –4

5 — 4 — 3 x

1-3 x

Теперь я могу решить обычным способом:

1–3x = 1
-1 -1
————
-3x = 0
— —
-3-3

х = 0

Является ли « x = 0» допустимым решением? Да, действительно, потому что ноль — допустимое число.Дело не в том, что решение — «ничто»; дело в том, что решение — это «что-то», а это «что-то» равно нулю. Итак, мой ответ:


Студенты, как правило, могут привыкнуть к тому, что ноль является решением уравнения, но разница между решением «ноль» (это решение является числовым значением) и «ничего» (возможно, является физической мерой чего-то вроде «без яблок» или «нет денег») может вызвать недоумение.

Убедитесь, что вы понимаете, что «ноль» сам по себе не является «ничем». Ноль — это числовое значение, которое (в «реальной жизни» или в контексте словесной проблемы) может означать , что «ничего» чего-то или другого нет, но сам ноль — реальная вещь; это существует; это что-то».


  • Решить 11 + 3
    x -7 = 6 x + 5-3 x

Во-первых, объедините одинаковые термины; затем решите:

Гм… подожди минутку …

С каких это пор четыре когда-либо равно пяти? Никогда! Есть ли какое-нибудь возможное значение x , которое «исправит» это уравнение, чтобы оно говорило что-то, что имеет смысл? Будет ли любое значение x когда-либо заставить это уравнение работать?

Нет; это просто невозможно. Я выполнил все свои шаги правильно, но эти шаги привели к уравнению (а) без переменных и (б) не имело смысла.Поскольку не существует значения x , которое заставило бы это уравнение работать, то это уравнение не имеет решения. Вот мой ответ на это упражнение:

.

Вот логика для приведенного выше примера: когда вы пытаетесь решить уравнение, вы исходите из (неустановленного) предположения, что на самом деле — это решение. Когда вы в конечном итоге получаете бессмыслицу (например, бессмысленное уравнение «4 = 5» выше), это означает, что ваше первоначальное предположение (а именно, что исходное уравнение действительно имело решение) было неверным; на самом деле решения нет.Поскольку утверждение «4 = 5» совершенно неверно, и с момента нет значения x, которое когда-либо могло бы сделать его истинным , то это уравнение не имеет решения.

Advisory: этот ответ полностью отличается от ответа на первое упражнение в верхней части этой страницы, где было , значение x , что будет работать (это значение решения равно нулю). Не путайте эти две очень разные ситуации : «решение существует и имеет нулевое значение» никоим образом не то же самое, что «никакого значения решения не существует вообще».

И не путайте приведенное выше уравнение типа «без решения» со следующим типом уравнения:

  • Решить 6
    x + 5-2 x = 4 + 4 x + 1

Сначала я объединю похожие термины; тогда решу:

Для предыдущего уравнения я получил «5 = 4», и не было значения x , которое могло бы сделать уравнение истинным. Этот результат противоположен этому. Существует ли для этого уравнения какое-либо возможное значение x , которое могло бы сделать приведенное выше утверждение ложным? Нет; 5 — это , всегда будет равно 5. Фактически, поскольку в последней строке вычислений нет « x », значение x явно не имеет отношения к уравнению; x может быть чем угодно, и уравнение останется верным. Итак, решение:

Это решение также может быть указано как «все действительные числа», «все действительные числа», «вся числовая строка», «(–∞, + ∞)» или « x ∈ & reals;» (последнее означает « x является членом набора действительных чисел»).Вы должны ожидать увидеть некоторые вариации в жаргоне от одного учебника к другому, поэтому не удивляйтесь различиям в форматировании.

Обратите внимание, что, если бы я решил уравнение вычитанием 5 из любой части исходного уравнения, я бы получил:

Другими словами, я бы получил еще одно тривиально верное утверждение. Я также мог бы вычесть 4 x с любой стороны, или я мог бы разделить обе стороны приведенного выше уравнения на 4, или я мог бы разделить на 4, а затем вычесть x с любой стороны, или я мог бы вычесть и 4 x , и 5 с обеих сторон исходного уравнения.Каждый из них — это еще один способ получить другой тривиально верный результат, например «0 = 0». Но независимо от конкретных предпринятых шагов результат (тривиально верное уравнение) всегда будет одним и тем же, и решение останется тем же: «все x ».

Поскольку (как я перечислил выше) существует много способов прийти к одному и тому же выводу для этого типа уравнения, вы не должны удивляться, если для уравнений «все действительные числа» или «без решения» вы не используйте те же шаги, что и некоторые из ваших одноклассников.Существует бесконечно много всегда верных уравнений (например, «0 = 0») и бесконечно много бессмысленных уравнений (например, «3 = 4»), также будет много способов (правильно) прийти к этим ответам.

Основным выводом из приведенных выше примеров должны быть следующие правила:

x = 0: регулярное решение регулярного уравнения

ерунда (например, 3 = 4): нет решения

тривиально истинно (например, 0 = 0): решение — все действительные числа

К сожалению, хотя вы почти наверняка встретите хотя бы один из этих вопросов типа «нет решения» или «все реально» в следующем тесте (и, вероятно, также в финале), их обычно не так много в наборе домашних заданий, и ваш инструктор, вероятно, предоставил только по одному образцу каждого типа.Это не дает вам большой практики в интерпретации решений такого типа, поэтому давайте еще несколько примеров.


Сначала я умножу 3 на скобку в левой части. Тогда я решу.

3x + 12 = 3x + 11
-3x -3x
——————
12 = 11

Моя математика верна, но результат — ерунда.Двенадцать никогда не будет равняться одиннадцати. Итак, мой ответ:


  • Решите 6 — 2 (
    x + 3) = –2 x

Я буду умножать и упрощать в левой части. Тогда я решу.

6-2 (x + 3) = -2x
6 — 2x — 6 = -2x
6-6 — 2x = -2x
0 — 2x = -2x
-2x = -2x
+ 2x + 2x
———
0 = 0

Ноль всегда будет равняться нулю, и в последней строке моей работы нет даже какой-либо переменной, поэтому переменная явно не имеет значения.Это уравнение верно независимо от значения x . Итак, мой ответ:


  • Решите 2 (
    x + 1) + x = 3 ( x + 2) — 2

Мне нужно будет умножить и упростить каждую часть этого уравнения.

2 (х + 1) + х = 3 (х + 2) — 2
2х + 2 + х = 3х + 6-2
2х + х + 2 = 3х + 4
3х + 2 = 3х + 4
-3x -3x
———————-
2 = 4

Нет; никогда не правда.


  • Решить 5
    x + 7 = 4 (2 x + 1) — 3 x -2

Мне нужно упростить правую часть, а затем посмотреть, к чему это приведет.

5x + 7 = 4 (2x + 1) — 3x — 2
5х + 7 = 8х + 4 — 3х — 2
5x + 7 = 8x — 3x + 4-2
5х + 7 = 5х + 2
-5x -5x
——————
7 = 2

Нет; никогда не правда.


Я разверну левую часть и решу.

8 (x + 2) = 2x + 16
8х + 16 = 2х + 16
-2x -2x
——————
6х + 16 = 16
-16-16
——————
6x + 0 = 0
—— —
6 6

х = 0

Это уравнение имеет значение решения, равное нулю.


  • Решить 1,5
    x + 4 = 4 ( x + 1) — 2,5 x

Я расширяю и упрощаю в правой части, а затем решаю.

1,5x + 4 = 4 (x + 1) — 2,5x
1,5x + 4 = 4x + 4 — 2,5x
1,5x + 4 = 4x — 2,5x + 4
1.5х + 4 = 1,5х + 4
-1,5x -1,5x
———————
4 = 4

Это всегда так, поэтому мой ответ:


Я разверну левую часть и решу.

2 (х + 5) = 2x + 5
2х + 10 = 2х + 5
-2x -2x
——————
10 = 5

Нет; никогда не правда.


URL: https://www.purplemath.com/modules/solvelin5.htm

Тем по алгебре: Решение уравнений

Урок 8: Решение уравнений

/ ru / algebra-themes / упрощающие-выражения / content /

Решение уравнений

В предыдущем разделе мы говорили о упрощающих выражениях .В этом разделе мы поговорим о решениях уравнений. Уравнения — это два выражения, равных друг другу с использованием знака равенства (=). Когда мы упрощаем выражения, наша конечная цель состоит в том, чтобы не осталось никаких операций.

Когда мы решаем уравнения, наша конечная цель — выяснить, чему равна переменная (или буква), поместив переменную отдельно по одну сторону от знака равенства и само число с другой. Мы собираемся достичь этой цели, выполнив два важных шага:

  1. Упростите каждое выражение по обе стороны от знака равенства.
  2. Используйте обратные операции для отмены.

Звучит сложно? Мы разберем его, чтобы было легче. Давайте посмотрим на пример:

5x — 4x — 6 = 18

Мы можем начать решать так же, как начинали бы упрощать выражение, проверяя порядок операций. Мы хотим максимально упростить каждую сторону знака равенства первые . Глядя на наше уравнение, нет скобок или показателей степени, и нет ничего, что можно было бы умножать или делить, поэтому мы просто начнем складывать и вычитать.Первая часть проста: 5 x — 4 x — 1 x , или просто x .

Отмена с обратными операциями

Теперь у нас осталось это уравнение:

х — 6 = 18

Мы не можем вычесть 6 из x , потому что они не , как термины (наш урок чтения алгебраических выражений объясняет это более подробно). Но x — 6 = 18 все еще недостаточно упрощен. В конце концов, мы ищем значение x , а не значение x — 6.

Чтобы решить это уравнение, нам нужно получить x только на одной стороне знака равенства. Чтобы переместить -6 на другую сторону от знака равенства, мы можем использовать , обратное — или противоположное — -6. Это будет 6. Другими словами, мы можем прибавить шесть к обеим сторонам уравнения.

В левой части уравнения -6 плюс 6 равно 0, а x -0 равно x . Справа 18 плюс 6 равно 24, поэтому x = 24.Теперь наше уравнение упрощено. Мы упростили его, используя , инверсию того, от чего мы хотели избавиться.

Это также называется , отменяющее , потому что оно позволяет вам отменить или избавиться от части уравнения. Это не значит, что вы можете просто вычеркнуть любую часть уравнения, которую не хотите решать (хотя это значительно упростит алгебру!). Вы должны соблюдать несколько правил.

Во-первых, вы заметили, что мы добавили 6 к в обе стороны нашего уравнения? Это потому, что две стороны уравнения всегда должны быть равными — в конце концов, это то, что означает знак равенства.Каждый раз, когда вы делаете что-то дополнительно к одной стороне уравнения, вы должны делать то же самое с другой. Поскольку мы добавили 6 к -6 на левой стороне , нам также пришлось добавить ее к 18 на правой стороне .

Во-вторых, помните, как мы прибавили шесть, где в исходном выражении говорилось, что вычесть ? Мы сделали это, потому что 6 — это противоположность -6. Чтобы отменить часть выражения, вам нужно использовать ее противоположную или инверсную. Противоположность вычитания — , сложение — и, как вы могли догадаться, противоположность сложения — , вычитание .

Посмотрите видео ниже, чтобы увидеть, как решена эта проблема.

А как насчет умножения и деления? Это тоже противоположности, и вы также можете их отменить. Например, как получить a только в этом уравнении слева от знака равенства?

5a = 30

Поскольку a равно , умноженному на на 5, вы можете разделить с обеих сторон задачи на 5. 5 a разделить на 5 равно a и 30 разделить на 5 равно 6, поэтому упрощенная версия этого уравнения будет выглядеть так:

а = 6

Посмотрите видео ниже, чтобы увидеть, как решена эта проблема.

Многоступенчатые уравнения

Давайте посмотрим на другой пример:

4 (2x + 3) = 68

Во-первых, нам нужно посмотреть, можно ли что-нибудь упростить. Помните, в предыдущем разделе мы говорили о числе вне скобок, означающем умножение? В соответствии с этим, мы можем умножить 4 · 2x и 4 · 3. 4 · 2x будет 8x , а 4 · 3 будет 12 .

8x + 12 = 68

Это дает нам 8x + 12 = 68 .

Теперь, когда обе стороны знака равенства упрощены, нам нужно будет использовать отмену, чтобы получить x отдельно. Прямо сейчас у нас есть две вещи, которые нам нужно переместить, 8 и 12. Мы добавляем 12, поэтому мы должны вычесть, чтобы переместить его. Мы также умножаем x на 8, поэтому мы будем делить, чтобы переместить его. Но какой из них двигаться первым?

Помните, что для отмены используется обратных — или противоположных — операций. Поскольку мы используем противоположные операции для перемещения объектов, мы собираемся использовать напротив порядка операций, чтобы решить, в каком порядке их перемещать.

Порядок операций гласит, что мы упростим умножение и деление перед сложением и вычитанием, поэтому мы собираемся сделать наоборот. Сначала мы будем использовать сложение / вычитание, а затем умножение / деление.

Сначала вычтем 12 с обеих сторон:

Поскольку 12–12 равняется 0, слева остается 8x. Поскольку 68-12 это 56, у нас остается 56 справа.

Наконец, разделим. 56/8 = 7

х = 7

Готово! Это означает, что для 4 (2x + 3) = 68 x должен быть равен 7.

Посмотрите видео ниже, чтобы увидеть, как решена эта проблема.

Практика!

Давайте попрактикуемся в том, что вы только что узнали, решив еще несколько задач. Помните, что для упрощения мы будем использовать порядок операций и , отменяя .

Обратите внимание на шаги, которые мы предпринимаем для упрощения этих выражений — через некоторое время у вас будет возможность решить несколько самостоятельно.

Задача 1

Упростите это выражение, чтобы найти значение x :

6x + 2 3 = 74

Найдите минутку, чтобы подумать, что бы вы сделали в первую очередь.Возможно, вы даже захотите достать лист бумаги, чтобы увидеть, как вы можете упростить это самостоятельно. Когда будете готовы, продолжайте читать, чтобы узнать, как мы получили правильный ответ.

Как и на предыдущей странице, мы начнем с того, что посмотрим, можем ли мы что-нибудь сделать с порядком операций . Это выражение имеет две операции: сложение и показатель степени .

6x + 2 3 = 74

Согласно порядку операций, нам нужно сначала вычислить показатель степени.Это 2 3 , что равно 2 ⋅ 2 ⋅ 2 , или 8.

6x + 2 3 = 74

Порядок операций гласит, что мы должны добавить следующее, но мы не можем добавить 6 x + 8 — переменная с коэффициентом, подобным 6 x , может быть добавлена ​​только к другому подобному члену. (Другими словами, число с переменной x может быть добавлено только к другому числу с переменной x .) Чтобы получить 6 x самостоятельно, нам придется отменить + 8.

6x + 8 = 74

Мы можем сделать это с напротив из 8, что равно — 8. Мы вычтем 8 с обеих сторон от знака равенства. 8 — 8 — 0. 74 — 8 — 66.

Мы почти закончили. Все, что осталось сделать, это избавиться от 6 из 6 x . Помните, что 6 x — это просто еще один способ записи 6 ⋅ x .

6x = 66

Поскольку 6 и x умножаются на на , мы можем сократить 6, сделав обратное: разделим .

6 x /6 равно x и 66/6 равно 11, поэтому x = 11. Готово!

х = 11

Как вы могли заметить, вам не нужно соблюдать порядок операций после того, как вы начали отмену. Все, что имеет значение, — это , при котором обе стороны выражения равны . Фактически, лучше всего отменить сложение и вычитание перед .

Задача 2

Попробуем другую задачу.Упростим и .

4 (3 года — 8) = 4

Эта задача немного отличается от предыдущей, но использует те же навыки. Вот как это решить:

В соответствии с порядком операций нам нужно сначала упростить выражение в скобках . Однако мы не можем вычесть 8 из 3 y — мы не можем вычесть число из переменной.

4 (3 года — 8) = 4

Поскольку 4 стоит рядом со скобками, мы должны умножить , указанное в скобках, на 4.(Запутались? Просмотрите наш урок по чтению алгебраических выражений).

4 (3y -8) = 4

4 ⋅ 3 y равно 12 y и 4 ⋅ -8 равно -32. Вы также не можете вычесть 32 из 12 y , поэтому для дальнейшего упрощения этого выражения нам придется начать отменять.

12лет — 32 = 4

Давайте сначала избавимся от -32. Противоположность -32 — 32, поэтому мы прибавим к обеим сторонам 32. — 32 + 32 равно 0, а 4 + 32 равно 36.

Мы почти закончили. Нам просто нужно отменить 12 из 12 y . Помните, что 12 y также можно записать как 12 ⋅ y .

12 y = 36

Поскольку 12 и y умножаются на на , мы можем сократить 12 на , разделив .

12 y /12 равно y , а 36/12 равно 3. Мы сделали это: y равно 3.

y = 3

Ваша очередь

Попробуйте решить следующие несколько проблем самостоятельно.Ответы ниже.

Задача 1

Упростите это выражение, чтобы найти значение x :

-2 + x / 5 — 3 = 0

Задача 2

Найдите значение y :

3 (y + 2y) = 36

Задача 3

Найдите значение r :

300–60 р + 10 2 = -380

Ответов:
  1. x = 25
  2. y = 4
  3. r = -2

Более длинные уравнения

Хотите верьте, хотите нет, но теперь у вас есть инструменты для упрощения многих выражений, даже таких сложных на вид, как это:

3x — 24 ⋅ 2 = 8x + 2

Это может показаться более сложным, чем задачи, которые вы решили на предыдущей странице, но вы будете использовать те же навыки, чтобы решить эту.Основное различие между этим выражением и другими, которые вы решили, заключается в том, что у этого есть переменная и по крайней мере одно число на по обе стороны от знака равенства , поэтому вам придется немного больше компенсировать.

Вам также нужно будет выбрать, хотите ли вы, чтобы переменная была слева или справа от знака равенства в вашем упрощенном выражении. На самом деле это не имеет значения — ответ будет одинаковым в любом случае, но в зависимости от задачи вы можете обнаружить, что математика кажется проще, чем в другой.Тем не менее, несмотря ни на что, ваше упрощенное уравнение должно иметь только переменную с одной стороны уравнения и только число с другой.

Давайте попробуем решить задачу вверху страницы: 3 x — 24 ⋅ 2 = 8 x + 2.

Во-первых, мы хотим разобраться в том, что мы можем, с порядком операций. Похоже, что все, что мы можем сделать, это умножить -24 ⋅ 2. Все остальное требует сложения или вычитания в отличие от терминов: — 24 ⋅ 2 равно -48.

3x -24 ⋅ 2 = 8x + 2

Давайте попробуем получить x на левой стороне знака равенства и цифре справа .Начнем с исключения -48 слева. Мы можем сделать это, добавив 48 к обеим сторонам. -48 + 48 равно 0, а 2 + 48 равно 50.

Поскольку мы решили, что x будет на левой стороне , мы должны избавиться от 8 x справа. Мы можем сделать это , вычтя 8 x с обеих сторон. 8 x — 8 x — 0, а 3 x — 8 x — -5 x .

Теперь все, что осталось сделать, это избавиться от -5 в -5 x . Поскольку -5 x — это способ записи -5 ⋅ x , мы можем отменить его, разделив с обеих сторон на -5. -5 x / -5 равно x , а 50 / -5 равно 10.

Готово! x равно -10.

х = -10

Как видите, упрощение этого уравнения на самом деле не было намного сложнее, чем упрощение любого из других уравнений в этом уроке — просто это заняло немного больше времени.

Посмотрите видео ниже, чтобы увидеть, как решена эта проблема.

Практика!

Теперь ваша очередь. Попробуйте упростить эти длинные выражения.

Задача 1

Решите для i .

-46 -2i = 42 + 7i ⋅ 6

Задача 2

Решить относительно j .

90j / 5 + 2 2 = 140 + j

Задача 3

Решить относительно k . (Подсказка: ваш окончательный ответ будет дробным.)

3 + (3k + 6k) = 3k + 5

ответов
  1. i = -2
  2. j = 8
  3. k = 1/3

Уравнения с более чем одной переменной

Иногда вы можете увидеть уравнение с более чем одной переменной, например, это:

2x + 6y -10 = 38

Если выражение содержит более одной переменной, вы не сможете упростить его полностью — недостаточно информации. Вместо этого в задачах с уравнениями с несколькими переменными обычно предлагается решить для одну переменных.Вы максимально упростите его, добавив переменную, которую вы решаете, с одной стороны уравнения, а любые другие числа и переменные — с другой. Упростим приведенное выше выражение: 2 x +6 y — 10 = 38.

Мы ничего не можем сделать с порядком операций, так что давайте начнем отменять. Нам нужно только x на левой стороне , поэтому мы постараемся расположить все остальное справа.

2x + 6лет — 10 = 38

Сначала отменим -10.Противоположность -10 равна 10, поэтому мы добавим к обеим сторонам по 10 . -10 + 10 равно 0, а 38 + 10 равно 48.

Далее избавляемся от 6 y . Мы вычтем из с обеих сторон. 6 y — 6 y равно 0. Поскольку с другой стороны нечего вычитать, мы просто напишем -6 y справа. (Смущает? Это как если бы мы вычли 6 y из ничего , или 0 — и 0-6 y равно -6 y .)

Теперь нам нужно избавиться от 2 из 2 x . Поскольку 2 x — это еще один способ сказать 2 ⋅ x, мы разделим с обеих сторон на 2, чтобы получить только x слева. 2 x /2 x , а (48-6 y ) / 2 — 24-3 y .

Это все, что нужно! Выражение не полностью упрощено — мы все еще не знаем числовое значение x и y — но оно достаточно упрощено, потому что мы можем сказать, что x равно 24 — 3 y .

x = 24 — 3 года

Помните, ваша цель при решении подобных задач состоит не в том, чтобы полностью упростить выражение, а в том, чтобы найти значение одной из переменных.

Это — это , которое на самом деле можно решить для двух переменных, если у вас есть более одного уравнения с одинаковыми переменными. Это называется системой уравнений. На самом деле мы используем системы уравнений в нашем уроке по задачам дистанционных слов, но мы не обсуждаем, как они работают в целом.Чтобы узнать больше о системах уравнений, посмотрите это видео от Khan Academy.

Посмотрите видео ниже, чтобы увидеть, как решена эта проблема.

Практика!

Задача 1

Решить относительно r .

88q + 4r — 3 = 5

Задача 2

Решить для с . (Подсказка: ваш окончательный ответ будет дробью со знаминателем r .)

(13ср) / 2 = 39

Задача 3

Решить для м .

6 м. — 30 чел. / 5 = 12

ответов
  1. r = 2 — 22 q
  2. s = 6/ r
  3. м = 2 + p

Проверка вашей работы

Важно проверять свою работу по алгебре, особенно когда вы только начинаете. К счастью, проверить свою работу, когда вы упрощаете уравнения, довольно просто. Все, что вам нужно сделать, это заменить переменную в уравнении на значение, которое вы нашли при упрощении.Чтобы увидеть, как это работает, давайте вернемся к одному из упрощенных ранее уравнений:

4 (3 года — 8) = 4

Мы обнаружили, что y равно 3. Посмотрим, правильно ли мы получили ответ.

Вот наше исходное уравнение. y — наша переменная, поэтому мы заменим ее найденным значением: 3.

4 (3 года — 8) = 4

Вот как выглядит уравнение с 3 вместо y . Теперь посмотрим, верно ли уравнение.Если левая сторона равна правой, наш ответ правильный.

4 (3 ⋅ 3–8) = 4

Мы будем следовать порядку операций, сначала скобки. 3 ⋅ 3 равно 9, а 9–8 равно 1.

4 (1) = 4

Теперь, когда мы упростили скобки, все, что нам нужно сделать, это умножить 4 на 1.

4 (1) = 4

4 ⋅ 1 равно 4. Обе части нашего уравнения равны, поэтому наш ответ правильный!

4 = 4

Это все, что нужно! Проверять каждое упрощенное выражение — хорошая привычка, и вы обнаружите, что проверка своей работы обычно занимает меньше времени, чем на упрощение уравнения в первую очередь.

Попробуем еще:

Выражение, на которое мы будем смотреть: 5 x + 3 = 23 + x . Мы проверяем правильность решения x = 4.

5x + 3 = 23 + x

Сначала заменим переменную x на 4.

5 ⋅ 4 + 3 = 23 + 4

Чтобы проверить нашу работу, нам нужно упростить обе стороны выражения. Начнем с левой стороны . По порядку действий нам нужно сначала умножить, а потом сложить. 5 ⋅ 4 равно 20, и когда вы прибавите к этому 3 , вы получите 23.

5 ⋅ 4 + 3 = 23 + 4

Теперь нам нужно упростить правую часть: 23 + 4 равно 27.

23 = 23 + 4

Наше уравнение не может быть правильным — 23 и 27 не равны . Теперь мы знаем, что x не равно 4. Другими словами, ответ неправильный .

23 = 27

Как вы только что видели, если вы проверяете задачу, и окончательное выражение — , а не — сбалансированное уравнение, ваш ответ — , а не правильно.Найдите время, чтобы вернуться и снова упростить исходное уравнение. Со второй попытки обратите особое внимание на порядок операций и убедитесь, что вы правильно складываете, вычитаете, умножаете и делите.

Хотите еще раз проверить последнюю проблему? На этот раз проверьте это с помощью x = 5.

Практика!

Задача 1

Проверьте эту проблему. u = 6 правильный ответ? Если нет, то что?

ед (3 + 8) / 2 = 33

Задача 2

Проверьте эту проблему.Правильный ли ответ против = 5? Если нет, то что?

В / 5 + 20 В = 19 В + 12

Задача 3

Проверьте эту проблему. w = 8 правильный ответ? Если нет, то что?

5 Вт + 3 = 4 Вт + 10

ответов
  1. Да, ответ правильный.
  2. №; v = 10.
  3. Нет; w = 7.

/ ru / algebra-themes / Introduction-to-word-tasks / content /

4. Решение уравнений

Помните такого рода задачи из начальной школы?

? + 5 = 7

Нам просто нужно было выяснить, какое число должно быть в поле, чтобы сделать его верным.Очевидно, нам нужно заменить вопросительный знак на «2»:

2 + 5 = 7

Решение уравнений с использованием алгебры ничем не отличается. Вместо поля мы используем букву для обозначения числа. Наша задача — найти правильное число (а иногда их может быть больше одного), которое делает уравнение истинным.

Иногда мы можем «увидеть» правильный ответ, если он простой (может быть, мы сможем просто сосчитать пальцами или что-то еще.) Но когда наши уравнения становятся более сложными, нам нужен процесс, чтобы следовать этому в конечном итоге даст нам ответ.

Наш процесс

  1. Мы стремимся получить x (или любую другую букву, используемую в вопросе) слева от знака равенства.
  2. Мы решаем уравнения по балансировке: что бы мы ни делали с одним часть уравнения, мы должны сделать то же самое с другой боковая сторона. Таким образом, если мы прибавим 4 к левой части, мы должны добавить 4 и к правой части.Если мы умножаем левую часть на 2, мы умножаем и правую часть на 2.

Пример 1

Решите уравнение

x — 6 = 10

Ответ

Нам нужно «избавиться» от -6 с левой стороны, чтобы у нас осталось x только с левой стороны.

Противоположность вычитанию 6 дает прибавление 6.

Если мы прибавим 6 к обеим сторонам, мы удалим -6 слева.

x — 6 = 10

х — 6 + 6 = 10 + 6

х = 16

Значит, значение x должно быть 16, чтобы уравнение было верным.

ПРОВЕРЬТЕ исходный вопрос:

16 — 6 = 10. Проверяется нормально.

Пример 2

Решить 5 x = 35

Ответ

На этот раз мы отвечаем

5 ×? = 35

Мы могли бы легко сделать это в уме (правда?), Но если проблема более сложная, нам нужно знать, что делать.

Слева мы умножаем неизвестное количество на 5. Мы будем использовать « x » для этого количества.

`5x = 35`

Противоположность умножению на 5 — деление на 5. Итак, мы делим обе части на 5:

`(5x) / 5 = 35 / 5`

Получаем:

`x = 7`

ПРОВЕРКА: 5 × 7 = 35. Проверяется нормально.

[Эти проверки кажутся глупыми с простыми примерами, но действительно хорошая идея для проверки ваших решений для всех задач с уравнениями, которые вы делаете.Это означает, что вы можете оставить проблему, чувствуя себя хорошо, потому что у вас есть правильный ответ, а также вы узнаете больше о том, как работает решение.]

Пример 3

Решить

`(3x) / 4 = 7`

Ответ

На этот раз нам нужно сделать 2 шага, чтобы решить уравнение. Мы замечаем, что внизу дроби стоит цифра 4.

`(3x) / 4 = 7`

Это эквивалентно делению на 4. Противоположность делению на 4 — умножение на 4.Итак, мы делаем это в первую очередь:

`(3x) / 4 xx 4 = 7xx4`

Отмена четверки слева дает:

`3x = 28`

На среднем шаге мы вычеркнули четверки, так что у нас не осталось дроби.

Теперь нам нужно разделить обе части на 3, так как у нас есть «3 ×» в левой части уравнения.

x = 28/3 = 9 1/3

Некоторые страны (например, США) оставляют ответ в виде одинарной дроби (28/3), в то время как практика в других странах (например, в Великобритании и Австралии) выражается в виде смешанного числа .

ПРОВЕРКА:

Наш ответ правильный?

Подставляя наш ответ в левую часть, получаем:

`(3x) / 4 = 3/4 x = 3/4 xx 28/3`

Отмена 3 (что дает нам 1) и 28 с 4 дает нам 7:

`3/4 xx 28/3 = 7`

Правая часть вопроса была 7, поэтому мы уверены, что наш ответ правильный.

Пример 4

Решить 5 — ( x + 2) = 5 x

Ответ

Сначала расширяем скобу.

`5 — (x + 2) = 5x`

`5 — x — 2 = 5x`

`3 — x = 5x`

Теперь мы понимаем, что легче разместить все x на правой стороне, добавив x с обеих сторон:

`3 = 6x`

Теперь я делю обе стороны на 6 и меняю местами стороны:

x = 0,5.

ПРОВЕРКА:

Мы проверяем наш ответ в обеих частях уравнения.Если это сработает, это должен быть правильный ответ.

LHS = `5 — (0,5 + 2) = 2,5`

RHS = `5 xx 0,5 = 2,5` = LHS.

Проверяет ОК.

Пример 5

Решить 5 x — 2 ( x -5) = 4 x

Ответ

Раскладной кронштейн:

`5x — 2 (x — 5) = 4x`

`5x — 2x + 10 = 4x`

`3x + 10 = 4x`

Вычитая `3x` с обеих сторон и меняя их местами, получаем:

`x = 10`

ПРОВЕРКА:

LHS = `5 xx 10-2 (10-5) = 50 — 10 = 40`

RHS = `4 xx 10 = 40` = LHS.

Проверяет ОК.

Пример 6

Если можете, решите уравнение

— (7 — x ) + 5 = x + 7

Что вы сделаете в заключение?

Ответ

— (7 — x ) + 5 = x + 7

Раскрыть скобки:

−7 + x + 5 = x + 7

Вычтем x с обеих сторон:

«-7 + 5 = 7»

Упростите левую часть:

`-2 = 7` ????

Это невозможно, поэтому мы заключаем, что для x нет возможных значений.

Добавить комментарий

Ваш адрес email не будет опубликован. Обязательные поля помечены *